PDF BCG Case Collection DD

You might also like

Download as pdf or txt
Download as pdf or txt
You are on page 1of 198

5.16.

Airline Company
Length: Medium (30 Minutes)

Problem:
Our client is a major airline company and the CEO contacts you (a BCG partner) and said
that he wanted to put one more passenger on each of his company’s flights. We will go to
a meeting with him now and we need to prepare two things:
• The financial return of one more passenger per flight. Is it worthwhile to do so?
• The marketing strategy to attract these new
n ew passengers.

Solution:
First element of the solution: The financial return
To calculate the financial return, the candidate should look that the revenue and cost side
of bringing one more passenger per flight.
One the revenue side, we need:
• Average cost of a ticket;
• Number of new passengers.
The candidate should also briefly ask about if he can assume that there is enough capacity
to put one more passenger onboard.
Second element of the solution: The average cost of a ticket
The candidate should ask about the kind of routes, the market share, and then price of
each of them.
In this case, we were talking about the Brazilian market and this airline has only domestic
flights. The interviewer simplified the numbers a lot the make the calculations easier.
on the city, we could have more frequent flights than others, but he agreed that I could do
this approximation. The interviewee can come up with other reasonable assumptions.

The candidate should also notice that normally the company spends a lot of money and
time performing maintenance on its airplanes. If the candidate forgets to mention this and
starts using all 110 airplanes, you should ask what assumptions he made to use the all the
planes at once. For this company, the interviewer mentioned that this company had a little
less than 10% of its airplane in maintenance.

So, assuming 100 airplanes available, we have:


• 100 × 80% = 80 airplanes doing 1 hour flights
• 100 × 20% = 20 airplanes doing 3 hours flights
To estimate the number of flights per airplane per day, we need the range of hours of
flights during a day and the interval of time in which an airplane stays on the ground.
What the interviewer wants to hear is not the right number, but what are your
assumptions to get to the numbers, for example:
Fourth Element of Solution: Flight time
• The first airplane departs at 6:00 am because the executives must arrive at its
client’s at 8:00 am, for example.
• There will be no flights after 10:00 pm, so we will assume that the last flight will
arrive at midnight.
So, there is no flight between midnight and 6:00 am.
Fifth Element of Solution: On the ground time:
• Taxing – 10 minutes
Case 11: R & D Productivity

BACKGROUND

Firm: BCG
Round: 2008 Summer, Second
Content: Qualitative

CASE QUESTION

Our client is the head of the R & D department of a pharmaceutical company.


co mpany. He wants
us to advise him on how to improve the productivity in the R & D depart ment

INTERVIEWER BRIEFING

Recommended approach: Explain a good structure to answering


answering the case. Suggest a
 possible framework and a solution.

Key facts:
Facts that are disclosed during the course
co urse of the case:
 There are basically several steps leading from the discovery of the molecule to the
launch.
Interviewee : The formula
formula would look
look like this:
this:

 NPV of future cashflows - R & D cost of


of successful drugs all drugs
R&D cost of all drugs

I nte
nter view
viewer : What are the big drivers to increase the productivity ?

Interviewe: Let’
Let’s look at each aspect o f the formula that we have written

 NPV of future cashflows


cashflows .
The big drivers of the NPV ofo f the future cashflows are as follows:
 Quality of the compound
Quality of the compound is the most important factor that influences the cash
flows. Say Lipitor for instance – 
instance – it’s
it’s the highest quality compound that’s out
there in the market and its capturing record revenues year after year. Its
important to ensure that sufficient tests have been done during the test phase
to ensure that there are no side effects to the drugs.
 Length of research
As it takes around 10 years to discover the drug, reducing the length of time
taken will increase the patent time thus maximizing our return
 Marketing & Sales promotion efforts
While this aspect is beyond the hands o f our client, its an important element of
other companies for promising drugs. This diversifies the risk as well as may
yield a good return

I nte
nter view
viewer :
 Any other ways to increase productivity? What about the R & D
organization?

Interviewee : We need to keep the scientists sufficiently


sufficiently motivated as it’s a very long
 process before their drug is successful.
successful. We need to ensure that they do not move
organizations quickly leading to loss of critical talent.

I nter viewe
viewer : What is the biggest challenge that the R & D head faces today?

Interviewee : The biggest


biggest challenge the R & D head
head faces today is on evaluation of
scientist teams. What kind of measurable metrics to set and how to evaluate the scientist
against it.

I nter viewe
viewer : What is the biggest mistake
mistake that an R & D head could make?

Interviewee : The biggest mistake


mistake an R & D head could make is to measure the scientists
scientists
 based on the number of drugs they pass from one stage to the next. This will motivate
them to pass certain drugs which maybe borderline or even not fit to be passed to the next
stage. This increases the R & D costs
co sts significantly
significantly and as mentioned before wastes time
that could have been spent on other products.
Case 13: Ice Cream Manufacturer

BACKGROUND

Firm: BCG
Round: 2008 Summer, Second Round
Content: Qualitative and Quantitative

CASE QUESTION

The client is an ice cream manufacturer. They are the European market leader with a
share of 31% of the ice
ice cream market. The client is
is concerned because its sales are more
seasonal than those of competitors,
competitors, even though overall profits are excellent. Note that
90% of his costs are fixed.

INTERVIEWER BRIEFING

Recommended approach: A good possible framework is to look at:

 Company
o Product mix (characteristics, seasonality,
seasonality, how perishable it is, etc.)
o Operations (is the manufacturing capacity used continuously or seasonally
o Very perishable products (by nature and regulation)
o Very extensive to store
 Operations
o Production follows seasonal sales pattern (too expensive to store)
o The production line could be used to produce other types of items.
 3 main types of Customers
o Ice cream stores (highly seasonal)
o Retail / supermarkets (moderately seasonal)
o Restaurants (almost
(almost not seasonal)

 Market situation

Channel Market split Client market share on Contribution to its


this channel overall market share
Retail / Supermarket 40% 40% 16%
Restaurants 30% 10% 3%
Ice cream stores 30% 40% 12%
Total 100% 31%

EXAMPLE DIALOGUE

I nter viewe
viewer :  How would you go about approaching this problem?
Interviewee: Maybe we could modify our products by adding preservatives and
stabilizers.

I nte
nter view
viewer :  Not a bad idea. However, regulations prevent us from doing this.

Interviewee: Ok. It seems that production of our ice cream will remain seasonal then. To
maximize the use of capacity and reduce our fixed costs burden, could the plants be used
to manufacture other types of products during non-peaks periods?

I nte
nter view
viewer :
This would be a possibility, but we have not explored it in details. What else
would you need to know?

Interviewee: We discussed the possibility to reduce the seasonality of operations. Now, I


would like to talk about reducing the seasonality of sales. Who are our current
customers?

I nte
nter view
viewer : Who do you think they might be?

Interviewee: As a customer, I can buy ice cream at supermarkets, ice cream


stores/chains, restaurants. Do we sell to all of these channels?

I nte
nter view
viewer : Yes, they are all our customers. Which do you think has the most seasonal
demand?
Interviewee: You mentioned initially that the total market share of the client is 31%.
Since retail/supermarket and ice cream stores contribute to 16% and 12%, restaurants
must contribute to (31-(16+12)=) 3%. Therfore, the client’s market share on restaurants
must be 10%. (30%/3%=10%)

I nte
nter view
viewer :  Based
on everything we discussed, what would you recommend the client do
to address its seasonality concern?

Interviewee: Firstly, I would recommend targeting more strongly the restaurants, as their
demand is less seasonal and the client is relatively weakly positioned there compared to
competitors. Secondly, I would recommend looking at products we could manufacture in
the current plants during non-peak times of the year, so as to optimize the use of the fixed
cost capacities.

I nte
nter view
viewer : Thank you.
Case 18: Liquor Company
BACKGROUND

Firm: BCG
Round: 2007 Summer, Second
Content: Mainly qualitative

CASE QUESTION

Our client is a multi-billion


multi-billion dollar liquor distributor. It has hired a consultant
consultant to
determine why its whisky brand is not growing.

INTERVIEWER BRIEFING

The key to this case is to understand the customer preferences are shifting away from the
medium range into either
either the high end or the low
low end. The high end provides the 70%
male segment with the refined taste and distinguished image and taste that they are
attracted to. The low end provides the 30% “bar“bar crowd” a cheap whisky to imbibe.
imbibe. The
rise in price has driven many of the customers
cu stomers away and the company needs tot o reconsider
its pricing strategy and branding strategy.

Competition and Market Share


Product

 Our client has increased the price from $11 to $12


 Competitors in the medium range are priced at $11
 Distillers have changed the ingredients to save $0.25/bottle on COGS but have
not changed the taste.

EXAMPLE DIALOGUE

I nterviewer:  How would you approach this case?

Interviewee: Well, I’d start externally: What has happened with the competition and the
customers? In terms of the competition, I would look at changes in market share, any
new competition, the segmentation, and market growth. Regarding segmentation, I
would also look at the customers, what do they want? Finally, I would look internally
and look at our product, marketing, and pricing.

I nterviewer: Great. Where do you want to start?

Interviewee: Well, what do we know about the market? How is it broken up?

I nterviewer:Well there is a high, medium and low end of the market? The high end has
10 competitors, the medium end has 2 including the client, and the low end has 4.
70% 30%
Male Younger males and females
40-55 years 21-35
Like taste, brand Bar and club crowd
Usually enjoys a drink after work Drink mixed
Into the tradition and brand
Drink whisky straight or on the rocks

Interviewee: That’s interesting. We must be rather traditional brand if we have such an


older following. Let’s turn internally. What do we know about our product? Have we
changed price, marketing, or the product recently?

I nterviewer: Our marketing has been consistent. The distillers recently changed
ingredients to save 0.25$ per bottle, but that hasn’t effected the taste. Also, we have
recently risen in price from $11 to $12.

Interviewee: That’s interesting. What is our competition priced at?

I nterviewer: The competition at the mid-range part of the market is at $11.

Interviewee: Well, that’s interesting. Do we know if this price change affected the
younger crowd?
Goizueta Consulting Association
Goizueta Business School, Emory University, 1300 Clifton Road, Atlanta GA 30322

Increase Profitability

Increase Profitability
- Revenues
- Price
- Elasticity
- Substitutes
- Volume
- Promotion
- Place
- New marketing campaign
- Economies of scope
- Cost of goods sold
- Material
- Scale
- Supply
- Inventory management
- Labor
- Consolidation of operations

Sample Case: Paper Towel Profits (BCG)


Interviewer
Your client is a large consumer goods manufacturer and is concerned about the paper towel
business. Profits for the business are okay, but the client wants to increase them. The client has
already done some research and following are the three findings:
Goizueta Consulting Association
Goizueta Business School, Emory University, 1300 Clifton Road, Atlanta GA 30322

Industry Firm Revenues Costs Product Sales


(Var)
Profitability Promotions Number of towels Fixed PLC/ Buying
Variability Share-of-voice Number of Variable Cycle
Competitors Scanner-based consumers - Manufacturing Frequency of
Frontloading? - Distribution promotions
Increase through Anticipation of
price reductions? promotions
New products? Distribution
I’d want to explore four broad areas: industry, firm, revenues and costs. Starting with the
industry, how is the profitability in the industry in general? And variability?

Interviewer
Good questions. But those are not the issues here.

Candidate
How many competitors are there?

Interviewer
There are three big competitors. All players own close to 25% market share each.

Candidate
Seems like industry factors are not important here….moving on to the firm…I’d be particularly
interested in finding out about the promotions. What is the share of voice? How are these
promotions run – are they scanner based, or just based on order quantities?

Interviewer
Goizueta Consulting Association
Goizueta Business School, Emory University, 1300 Clifton Road, Atlanta GA 30322

not on the items they buy. This will prevent frontloading, and will encourage th e retailers to
promote sales by passing on some benefit to the consumer or providing us some display space
etc. Finally, I think we can reduce fluctuations by strengthening the brand. That will make the
retailers feel more secure.

Interviewer
That sounds impressive. But how will you convince the retailers to agree to this?

Candidate
I think the best way to convince the retailers is to make sure that they will earn more, and to
make them aware of the fact. For instance, buying in advance necessitates that they warehouse
the product. By making the system more “on demand” and less dependent on warehousing, we
are saving them money in inventory costs. Also, we may help the retailers put the technology in
place for the operation. This can be used for other products as well, saving them even more
money. Besides, if the promotions are scanner-bases, they will focus on selling more, giving the
product display etc. which will increase their reven ues.

Interviewer
Cool. This is going well. Let us assume that scanner promotions will actually save money. Let’s
consider 5,000 units
• Scan  Costs are those of 5,000 units

• Bill-back (post facto educated guess)  Costs for those 5,000 units are equivalent to the costs

for 7,000 units under the scanner-based method.


• Off-invoice (before fact educated guess)  Costs are equivalent to those of 10,000 units.

Given that the profit per unit is $15 and the cost per unit is $10, how do the return on
investments for these methods compare?
Goizueta Consulting Association
Goizueta Business School, Emory University, 1300 Clifton Road, Atlanta GA 30322

BCG - Trevor's Toys


You have been assigned to work on a product-offering strategy case for Trevor's Toys, Inc. Your
team has been asked to deliver a recommendation quickly: you have only 12 weeks (or 60
working days) to complete the project. You must use your time wisely if you’re to deliver
maximum value for Trevor's Toys.

Trevor's Toys is a regional toy retailer focused on the high-tech toy market in the United States.
It generated $600 million in revenue last year from 30 stores located primarily in urban areas
throughout the Northeast and the Midwest. Management has considered expanding its product
offering to include low-tech toys a number of times, but has never been convinced that it was
making a fully informed decision. Revenue growth for the company has historically been strong,
although it has slowed in the last few years.

Trevor's Toys' competitors include traditional toy stores and online Web sites that sell both high-
tech and low-tech toys, some of which are quite large and utilize both sales channels.
Management is concerned that the continued growth of the large traditional stores and online
toys sites will erode its customer base over time unless Trevor's offers a full selection of to ys.

Your task is to analyze the decision facing management at Trevor's Toys. Your first step is to
frame your analysis, decide which aspects of it are most important, and then ask questions about
the most relevant topics. Afterward, you'll develop a recommendation and a rationale to support
it.

Doing the analysis begins with framing the issues. This allows you to establish the scope. By
doing so, you'll ensure that the questions you select have real impact, and that your attention is
Goizueta Consulting Association
Goizueta Business School, Emory University, 1300 Clifton Road, Atlanta GA 30322

Process re-engineering
Implementation
Organizational effectiveness

Assignments can be framed in any number of ways. The key is to use a logical structure that
enables you to analyze and respond to the issues facing the client. Although the four issues
below may not be the ones you selected, we recommend that to achieve the greatest impact for
the Trevor's Toys management team, you thoroughly investigate them during the time you have
remaining.

Customers
What are the needs of the client's customers, and how would a broader selection of toys address
those needs? Will a move to expand the product offering add value to the customers' experience?

Competitors
How will the current competitors in the traditional toy space react to the decision by Trevor's
Toys to offer low-tech toys?

Market opportunity
Is the market large enough to support a profitable Trevor's Toys venture into low-tech to ys?
Would Trevor's capture enough share for this to be successful?

Implementation
Does Trevor's Toys have the resources and capabilities to increase its product offering? What
would be required to make that happen?

As with almost all project work, you won't have the time to do as much analysis as you would
Goizueta Consulting Association
Goizueta Business School, Emory University, 1300 Clifton Road, Atlanta GA 30322

• What are the attributes of Trevor's Toys' customers? 3 days


• What are customers looking for in an ideal toy store? 4 days
• Would customers do all of their toy shopping at Trevor's if the company were to offer both
low- and high-tech toys? 3 days
• How loyal are Trevor's customers? Are many sales repeat sales to the same customers? 3
days
• How much time is required for an average toy shopping trip in a traditional store (ours,
competitors')? 2 days
• Who are Trevor's Toys' primary customers shopping for? 1 day
• Do customers shop online for toys? Why? For what kinds of toys? 1 day
• How do they decide if they are going to purchase online? 2 days

Questions: Competitors
• Who are Trevor's Toys' traditional competitors? 2 days
• Who are the market share leaders in the total toy market? 1 day
• What are the different product offerings among online competitors? 2 days
• What are the competitors' growth rates? 1 day
• What do competitors offer that we can't? 3 days
• Are the competitors making any profit? 1 day
• What are the competitors' pricing policies? 2 days
• How are the competitors promoting and marketing products? 2 days
• What is the selection of low-tech toys available at the competition? 2 days
• What is the selection of high-tech toys available at the competition? 2 days
• Other policies affecting customers (returns, customer service, etc.)? 2 days
• What sites for online toys attract the most traffic? 1 day
Goizueta Consulting Association
Goizueta Business School, Emory University, 1300 Clifton Road, Atlanta GA 30322

• How fast has the toy market grown over the past five years? 1 day
• How fast has the high tech toy market grown over the past five years? 1 day
• Why is the toy market growing so fast? 2 days
• What is the average amount spent per child? 2 days
• What is the size of the online toy market today? 1 day
• What is the anticipated growth for the online toy market? 2 days
• How much of online spending is expected to be for high tech toys? 2 days

Questions: Implementation
• Can the current facilities be used for the sale of low-tech toys? 2 days
• How does management at Trevor's To ys view the low-tech business as a competitive threat?
3 days
• Are the suppliers of low- and high-tech toys different? 2 days
• Where would Trevor's Toys source the low-tech toys? 2 days
• What inventory system does Trevor's currently use? 1 day
• What is the organizational structure at Trevor's Toys? 3 days
• Would the current internal culture fit the venture into low-tech toys? 3 days
• How will Trevor's Toys advertise the change? 2 days
• Are there any international trade issues? 1 day

Would customers do all of their toy shopping at Trevor's if the company were to offer both low-
and high-tech toys?
Although many customers would purchase both types of toys at Trevor's if they were available, it
is unlikely that customers would stop going to traditional toy stores. Given their wider selection
of toys, larger size, and lower prices, those stores will co ntinue to be a major draw for many of
Trevor's customers.
Goizueta Consulting Association
Goizueta Business School, Emory University, 1300 Clifton Road, Atlanta GA 30322

Most of Trevor's customers are fairly loyal to the compan y and visit the store, on average, three
times a year.

Do customers shop online for toys? Why? For what kinds of toys?
Yes, a number of customers shop online. For the most part, people purchase online for the
convenience of not having to visit the store. The amounts of high-tech and low-tech toys sold
online are roughly equal and are expected to remain so for the next few years. An exit survey at a
number of Trevor's stores revealed that only 20 p ercent of Trevor's current customers shop
online.

Who are Trevor's Toys' traditional competitors?


The biggest traditional store competitors are Toys R Us and the large discount stores, like Wal-
Mart, J.C. Penney, and Target. They each have hundreds of stores nationwide. Each of these
stores sells a small selection of very competitively priced SKUs, which T revor's Toys also
carries. Trevor's Toys differs by offering a wider product range and narrower geographic
coverage. Trevor's Toys is located only in the U.S. Northeast and Midwest. No one else offers
the same variety and depth of product.

What do competitors offer that we can't?


The large competitors offer a great variety of to ys in all product categories. The competitors also
have national coverage. Currently, Trevor's Toys does not serve customers outside its store
regions.

What are the competitors' growth rates?


The large stores (Toys R Us and Wal-Mart) have been seeing 3 percent and 7 percent growth in
this category over the past few years. The pure-play online toy retailers have been growing at
very high rates of between 75 percent and 150 percent per year since they began operation in
Goizueta Consulting Association
Goizueta Business School, Emory University, 1300 Clifton Road, Atlanta GA 30322

Change the standards – alter the standards on our new phones and switches every couple of years
to make old phones incompatible with them. This strategy would hurt the used vendors, but
might tick off our customers as well.

Buy back our own used equipment – this is a possibility, but it’s also very expensive. A variant
on this approach might be offering trade-in for a customer’s older, used phones.
Spin off a lower line new product – this will make our new product more price competitive with
the used product. The key here is trying to find what it is that customers like about the used
product, and trying to match this with our new products (i.e., price, delivery speed and frequency,
customer service, etc.).
Goizueta Consulting Association
Goizueta Business School, Emory University, 1300 Clifton Road, Atlanta GA 30322

Automobile Market-entry (BCG)


Your client is a joint venture between two European (German) companies (later learned this to
be Mercedes-Benz and Swatch) who have developed a new automobile. The car was designed
for the European market, but your client would like to know the viability of introducing the car
in the U.S. market. The car itself is 98 inches long (2.5 meters), has two seats, two doors, a 55
horsepower engine, and gets excellent gas mileage. It is very small, low cost, and its design was
driven by new technology. For example, this car is the first of its kind to offer detachable body
parts to facilitate easy changing of car colors, and its interior micro-electronics technology is
state-of-the-art. The car comes in two varieties: a regular hard-top version that would come
fully loaded for about $11,000 US dollars, and a convertible rag-top that comes similarly
equipped for $14,000. For purposes of this discussion, consider only the introduction of the
hard-top version. Your client would like three questions answered:
1) How would you segment the U.S. automobile market?
2) Which segments would this car most likely address?
3) How would you handle distribution?

Response

Question 1
Car customers can be segmented in a number of ways. For example: income level, sex, state of
life (i.e., married without children, married with children, single, retired, etc.), age, geographic
location (i.e., live in a city, a neighborhood, in a rural area), according to the criteria the
consumer uses to make his car purchase decision (i.e., sex appeal, styling, features, price,
practicality, etc.), or simply by type of car purchased (i.e., sport utility, sports car, 2-door sedan,
4-door sedan, wagon, etc.). Probably does not matter which angle the candidate takes, but he
Goizueta Consulting Association
Goizueta Business School, Emory University, 1300 Clifton Road, Atlanta GA 30322

quality of engineering, styling, performance and reliability. The brand is top-notch in the states
(let the candidate dig for this, because this is the most important part). The dealerships would be
a nice distribution medium for the client. But what about the translation of the brand name.
Wouldn't this low end car damage the existing high end reputation of the existing brand
(probably)? What about the price elasticity of demand of the two consumer segments (high end
versus low end)? High end buyers are more price inelastic (they purchase on performance, brand,
etc.), while the lower end buyers are more price elastic (they purchase on price). How will our
existing lattice of dealerships do in marketing such a different product to such a different target
market? Probably the only viable solution would be to create a new brand name and a new
network of dealerships, or at least let another type of dealer sell the cars for the client. So, brand
transferability is critical to the question of distribution. By the way, BCG advised the client not
to enter the U.S. market, primarily for the reasons discussed here.
Goizueta Consulting Association
Goizueta Business School, Emory University, 1300 Clifton Road, Atlanta GA 30322

U.S. Photographic Industry Re-entry (BCG)


Your client is a manufacturer and distributor of specialty photographic products in the US. The
products include all essential materials used to make the printing of a newspaper or magazine
happen, including but not limited to: imaging plates, film, specialty lights, and developing fluids.
The client is the #1 provider to newspapers nationwide. However, the client does not have a
presence in the commercial market. The commercial market includes magazines, brochures, and
any other type of glossy publication printed in the US. The client has already failed once in
penetrating this market, and would like help developing its re-entry strategy into this profitable
market. Why did the client fail the first time, and how should he proceed this time?

Response

Competitive Landscape
There are 5 large competitors. We lead the Newspaper Market with about 40% share, the other
four have about 15% each. The other 4 split the Commercial Market, with about 25% each. We
have none.

Company (Strengths and Weaknesses)/Customer (Market)


Start off trying to understand what makes the company tick. Since it is extremely successful in
the Newspaper Market, but has failed miserably in the Commercial Market, the candidate should
try to understand what has made the client successful in the former, and what are the differences
between the two markets. Basically the company has succeeded by manufacturing the highest
quality product in the Newspaper Market, and by developing a set of relationships with the 300
major clients. So, it's success has been based on quality of product, and distribution through
superior relationships. This should lead the candidate to further questions....
Goizueta Consulting Association
Goizueta Business School, Emory University, 1300 Clifton Road, Atlanta GA 30322

In addition, the quality required to satisfy Commercial clients is different. The nature of the
printing process requires more exact print types, requiring more intricate ph otographic products.
Our client is not prepared to satisfy these requirements.

What is the cost of this quality?


Tell the candidate if he probes here that to achieve this quality he must lay out $6M in machinery
investment. Ask him whether this is a worthwhile investment. The candidate could talk about
the size of the market, and different competitor's share, and hypothesize what the client could
expect to capture given this investment. I didn't get specific numbers, but just talked in terms of
using WACC to pay off these fixed assets with a projected stream of future earnings in this
market. This analysis lets the candidate show off some finance competence. Also, he could talk
operationally about different types of quality (performance vs. conformance), and the cost of 6
sigma (or some other level of) quality, and the ramifications that quality has on the process, labor,
the culture, and the bottom line.

What other differences prevent the client from succeeding in the Commercial Market?
Let the candidate struggle over this problem, but the primary problem is a tip you gave earlier.
The Commercial Market is characterized by many smaller orders, while the Newspaper Market
by fewer, long-standing high volume orders. There are only about 400 major newspapers in the
US, and our client's sales force of 75 has great relationships with all of them. In the Commercial
Market, there are over 34,000 clients nationwide, touched by a lattice of about 1,200 dealers. So,
distribution is different. Our salespeople can't touch everyone so easily in this new market. Also,
the Commercial Market is characterized by long standing orders with each of the clients.
Usually a client will select a photographic product vendor and stick with him for 5 to 7 years.

Is this an opportunity or a threat?


Both. It is an entry barrier, which prevents our client from accessing all new business in the first
10. C��� ��������

10.1 D������� �������� ���� (BCG)

���� 1: A������� ������ �� ��� ����

Y��� c����� �� ��� �a����� ���c���� ���a���� �� Ca�a�a, ���� 500 ������ ����a� ���������� ���
c������. L��'� ca�� �� Ca�a�aC�. F�� �����a� ��a�� �������, Ca�a�aC� �a� ����a���� ���
��c�����a����� Ca�a��a� ���a���� (300 ������) �� b��� ���a���� �a���� ��a�� a�� ������ab�����.
H������, ��� �a����� ���c���� ���a���� �� ��� U����� S�a���, USC�, �a� ���� b����� ���
Ca�a�aC�'� c���������� a�� �� ��a����� �� c������ a�� 300 ������ �� USC� ������. T�� CEO ��
Ca�a�aC� �� ����� ������b�� b� ���� ���� �� ������, a�� a��� ��� ��� ��������� ���������:
S����� I b� �������? H�� ������ I ��ac�? H�� ����� ��� a����� ��� CEO?

���� 2: E�������� ������������� �� ��� ����

��, ��� ������, C�����C�, �� ������ ����������� �� C����� ���� � �.�. ����������. O�� ���� ��
�� �������� ��� ������ �� ��� ������ ��� ������ ��� ������ �� � ��������. B����� I ��� ������
��� CEO I ���� ���� ���� ����������� ����� ��� ���������. F���� �� ���, I'� ��� ���� I
���������� ���� � �������� �������� ��!
T�� ������ a�� ��ca��� �� �����a� �����a���c �������. I� �ac�, ��� ����� ���� ��� a Ca�a�aC�
����� �� ��� c�����, a�� ��� c���������� �� ��� ���� ���� c�����.

D� C�����C� ��� ��� ����������� ���� � ������� ������� ���?

Y��. Ca�a�aC�'� ������ ���� �� �a�� a ����� �a����� �� b�a�� �a���, b�� b� a�� �a���, ���
�����c� ��� �� �����a�.

 A�� C�����C�'� ������ ������������� ����� ���� ��� �����������'�?

N�. F�� c���a�� ����� Ca�a�aC� �� ���� ���������, a�� ��� ������ ��� c���������� �� ����
���������, b�� ��� a���a�� ���c� ����� �� �����a�.

I� C�����C� ���� ���������� ���� ������� �� ��� ���� ������, �� ���� �� ���� ������ �������
 ��� �����?

I� ac��a��� �a� ������ ������� ��a� ��� c���������� �� a ��������� ba���.

����, ������ ������� ����� �� ��� ������ �� ����� ����� �� ������ ��������. A�� ��� ������ ����
����� ������� ��� �� ����� ����� ���� ��� �����������'� �� ��� ������ �� ������ ��������� �����?

Ca�a�aC�'� c��� ����c���� ���'� a�� ����� ��a� ��� c����������'�. I�� ������ ��������� �������
a�� ��� �� ������ ��������� �a���.
I� ���� ����, I ����� ����� ���� ��� C�����C� ������ ��� �������� ������ �������, ����� ���
���������� ����������� ���� � ������� ��������� �� �������� ������.

Y�� a�� ��ac��� �����. I� ����� ��� ��a� Ca�a�aC�'� ������ �a��� a�� ��� ����a���� �� a
�������ca���� ������ ����� �� c������� �����c�. T�� ������ a�� c��a���, ���� a���ac����, b�����
���c���, a�� �� ��. T�� c���a�� ���c������ ���� ������� a ������ �� c������� ������� �a��
��a�. I ����� ���'�� �����c������ c������ ��� Ca�a��a� �a��������'� ���� ��� �� a ���c������
�� ��� U.S. �a����.

H�� ���� ������ ���� ��C� ��� �� ��� �.�., ��� ��� ���� ���� ��� ��������������
�������� �������� ���?

USC� ���� 4,000 ������ a�� ��� ��c�����a����� c��������� ���� a�������a���� 1,000
������.

 A�� ��C� ������ ������ ���� ����� �� ��� ������� �������� �������� �� ��� �.�.?

Y��. USC� ������ a���a�� 200,000 ���a�� ����, �����a� ��� ����ca� ���c���� ���a�� ����� ��
a�������a���� 100,000 ���a�� ����.

����� ������� ������� ���� ��C� ������ �� ������� ������� ����� ����� ��� ������ �� ���
������� �.�. ����������!

C����. USC�'� �a��� a�� a�������a���� $5 b������, �����a� ��� ��a���� c��������� ����� ab���
�����. A� �� ���� a�� ��������� ���� c���� ���� ���������, �� b��a� �� b�� ��� �����
���c���� ���a����� a�� c������ ����� ������ �� ��� USC� ����a�.

�� �������� ��C� ���� ��� � ��������� �����, �� ���� ���������� ������� ��?

N�� ��c���a����. S�������� �� ����, b�� ���� I �a�� �� c������� �� �� ��� USC� ����a�, I
��a�� ��a� �� ca����� ��� �a�� b�a��� a� ���c�� ��a� a�� �� a���a�� ��� ���c��� ����� ��a�
��� c����������'�.

���� �������� ���� ��C� ��� �� �������� ������� �� ������ ���������� ������ � ����� ��'� ���� 
������ ���?

I� ������� �� a ��� �� �ac����, ��c� a� ��� ���� �� ��� �������� �����, ��ca� �a����
c����������, ��ca� ��a� ���a�� c����, a�� �� ��, b�� I ���'� ����� �� ���� �� �� ���� ��a�
����.

����, I ������� �� ����� �� �������� �� ����� �� ���������� ���� �� ���� �� ���� ��� 300 ������
���� �� ������ �� C�����.

L��'� ���� a����� ��a� �� �����'� ��a� �� ���a�� ��� Ca�a��a� ������ b����� ����� c������
����.
OK. I ����� I'�� ������� ������ ����� ��C�. I'� ���� �� ��� � ��� ��������� ����� ��C�'�
������� �� ������� �� ��� C������� ������. D��� ��C� ���� � ������ ����� ���� �� C�����?
Ca�a�a ���� �a�� �������ca���� ������ �ab�� c����, a�� I'� ��� ���� ab��� ��� c���� �� ��a����
��ac�. W�a� a�� ��� ������� a�?

I ��� �������� ���� �� ����� ���� � ������ ���� �� ����� �������� �� C�����, ������� ��C�
����� ���� �� ������ ������ ������ ���� �� ���� �� ��� �.�. �� ����� ��� �����.

T�a�'� ���bab�� ����, b�� �����b��, Ca�a�aC� ���� a��� c��� ���� ��� �a�� ���� �ab��
c����. Ca� ��� ����� �� a�������a� c���� ��c����� b� USC�'� Ca�a��a� ����a����� ��a� �����
��� b� ��c����� b� Ca�a�aC�?

��C� ����� ����� ������ ������������ ����� ���� C�����C� ������� �� ���� ���� �� ����
 ������� ���� ��� �.�. ���������� �� �� C�����.

Y�� a�� �a���a��� �����. Ca�a�aC� �a� ��� a��a��a�� �� ������b����� c����, ���c� ��� �������
��a�� ���� �����a���c a��a a�� �� ���� ���� �����c�� ���� Ca�a��a� ���������. H������,
���c� Ca�a�aC� c�������� �� ��� a ���� ��a� �� �����c�� ���� ��� U.S., ��� ac��a�
a��a��a�� �� Ca�a�aC� �� ��� ���a������ ab��� ��� ���c��� �� ����a�� c����.

 A�� ���� �������� ���� ��C� ���� �� ���� �� ������ � ����������� ����� ��������� ����
C�����C�'� ������: �� ��� ��� �������, ���� �� ����� ����� �� ����� �������.

I ����� a���� ���� ��a� c��c������.


����, ��� ������ �� ��� ���� ���������� ��� ��� �������� ��� � ���� ������.

F��a���, �� ����� ���� ��� �a��� ��a��� ���� ��� �a�� �� �����; ��� �a� ���a���� ac������'�
a���ac�������� �� �� ��a� ��a� ��� ����� �a�� �� b� ������ �� ������� �� �� a� ��� �������!
B�� ��� ����� �� b� a b�� ����a�� �� ���� ��� c����� ��� �� �����? (C��a������)

�� ��� ��� ������� �� ��� ����� ��������� �������: ��� ��� ������ ���� ������� �� ���
����������� �� ��� ����������� ���� ��� ��������, �� ���� �� ��� �������� ����������. ��
���� ���� ���������� �� ��� ������'� ���������� ����.

I� �ac� ����� �� a b�a� �� ��� ��a������ ���� ��� ��a��. W�a� �� ��? (C��a������)

�� ���� ������ �� ��� ��� ������ �� � ����������� �����. B�� �� ���� �� ���� ���� �������
���� ��� ����� �� ��� ���� ����� ������ ��� ���� ����� ������ ��������� ����� ����� ���
������. ����� ��������� ����� ����� ���������� �������� �� ��� ��� ������: �����������, ���,
�������� �� ����� ��������.

A�� ����� ����� ������ ��a� ����� ��ab�� a ��a��� �� ����� ��� �a� �a���� �� a ������ab��
�a�? (C��a������)

B� �������� ����� ������ �������� �� �������� ��� ��������, ����� ��� �� ��������� ���� ���
��� ������:
C������ ���������/�������� �����
M������ ���� ����� �������� ��� ����� ���������� ����
10.5 C������� AD�� �������� ����� (BCG)

��� ��������� �� �� �������:

ADSL �� a ��c������� ��a� ��ab��� ��� ���������a���� �� b��a�ba�� I������� �����c�� ��a
��� �������� ��������� ����a����c����. T��� ��������� ����a����c���� �� ����� b� ���
��c��b��� ����c�� ����a��� �� N���a�. A� �� ���� E�����a� c��������, ��� �����a��� �a�
����� ��a� ��� ����a��� �a� ����� ADSL �����c��, ����� ��� �������� ��c��b���������
��������� ����a����c���� ��a MDF acc���. T��� ��a��, ��� ��� ����a�� ca� ���� ��� c�����
���� ���� ��� ��c��b��� ����a��� b�� �a� �� ���c�a�� ��� ��� ADSL ���������.

T�� ��c���ca� ������ ��a� a ��� ����a�� ����� ���� �� ����� �� ���ab���� a� ADSL
c����c���� ba��ca��� c������� �� ����� ��������: MDF acc��� (c����� ����), ADSL ���������,
I������� ������ ca�ac��� (��b�� acc��� c����c���� ��� ADSL ��������� �� ��� I�������).

T�� c���� �������� �� ���ab������� ��� ��c���ca� ������ a�� ����� (a������ �a�� b���
c�������� �� E����):
MDF acc��� �a���� �� EUR 12 ��� ���� ��� �����, ��� b� ��� �����a���
R������� ����������� ��� ��� ADSL ��������� �� EUR 120,000 ��� ��ca����, �����c�a����
������ �� 5 ��a��. F�� ������c��� ��a���� �� a����� ����a� �����c�a���� ���� ��
�������� c����. I� ���a�, ����� a�� 250 ��ca����� ����� ADSL ��������� c���� b�
����a����, c������� a�� ���������� �� N���a�
I������� ������ ca�ac��� �� c�����c�a��� a�a��ab��. D�������� �� ��� �������� ��������
�����, c���� a�� �� a���a�� EUR 2 ��� �������� ��� �����
I: S����� ����, ��� ����� ��� ca�c��a�� ��� b��a������ �����?

C: H���, �� ���� ��� ��� �����, ��� �� ����� ���� ��� �������, ����� ����� �� �����
����� ������ �� ���������. �� I ����� ������ � ����� ��� ���� ��������� ��� ������ ��
��������� �� ����. ������������, I ����� ���� �� ����� �� ��� �������� ������ ��
��������� ����� �� � ���������� ������. D� �� ���� �������� ����� ��� �����?

I: W� ���� ��a� ��� ��c��b��� c�a���� �� a���a�� EUR 20 ��� �����.

C: OK, ����� ������ �� ����� ���� ��� E�� 20 �� ����� ����. L���� �� ��� ���� �� ����������
���� ��� ��� �� �� ���� �� ����� ��� ����� �� ����� �� ������ ���� �����������. �� ����� ��� ��
�� ��� ��������� ����� ����, ����, I ��� �� ���� � ��� �� ����������� �� ��������� �����,
��� I ����� ��� �������� �� ��������� �����, �� ���� �������?

I: W��� �������, �� a��� ���� a� ���a���a���� a�� a ��a�� �a������� b�����. T�� ���
����a�� �a� ���� ���� �������� a�� �a� ca�c��a��� ��a� a� a���a� b����� �� EUR 4.8
������� ����� b� �����c���� �� c���� ��� ������ c������.

C: OK, ���� I ��� ��������� ��� ���������� �����. P�� ��������, ��� ��� ������� ����� E��
20, �� ����� �� ���� E�� 14 �� ����� ��� �������� ��� MDF ������ ��� ��� I������� ������.
���� ����� �� ��� E�� 6 ��� �������� ��� ����� �� ����� ��� ����� �����. �� ����� ��� ���
 AD�L ��������� �� 5 �����, �� ������ E�� 2,000 ��� ����� ��� ��������. F�� 250 ���������
������ E�� 500,000 ��� �����. I� ��������, �� ���� �� ����� ��� �������������� �����, �����
C: OK� �� ������ � ���� ������� ������. I ����� ��� �� ����� �������� �� ��������� ���
��������� ���� ��� ����� ���� �� ��������� ����� �������, ����� ���� �� ��� ������� ����� ���
����� �� �� �������� ������ �� ������� ��� ��������� ���� �� �������� ��������� ���� ���
����������� �� ������. N��, �� �� ����� ���� � ��������� ��� �������� 5 ����� ����� ��� �� ��
������ ���� �� ���� ��� ������ ��� ����� ����� ���� ������� ��� ���� ������, ��� AD�L
������ ��������� ���� 20% �� 40%, ����� ����� 400,000 ��� ���������� ���� ��������� ��
 AD�L. �� ����� �� ��������� ��� ����� �������, ������� ��� ��������� ��������, ��� �����
���� ����� ��� �������� �� ��� ���������. ��� ����� ��� ��� ��������� �������� ���� ��
����� � 50% ����� �� ����, ����� �� ��� � ������ �������� �� ��� ������ �������. L���� ������
�� ����� �� ��������� ������� 1 �� 2 ����� �����������, ����� ����� �� ����� ��� 17�25%
����� �� ��� ��� ���������, �� �70,000�100,000 �� �����. �� �� ����� ���� �� ���'� ��
 �������� �� ����� ���������� ���� ��� ��������� ����. �� ����� ���� ���� � ���� �� ���
�������� ���������, ���� AD�L ��� ����� ���������. B�� ���� I ����� ���� �� ���� ���
������ �� �� ���� ���� ����� ������?

I: OK, ��a� �� ��������� ��� �a� �a�� �� ��������a��. B�� ��a� ����� b� ���� �������
 �������� �� ����?�

C: I ��� ������� ���� ��������� ����� ���� �� ������ �� ���� ��� ���� ����������� ���� ���
������� �������, �� �� ��� ��� ������� ����� ����� � ���� ����� �����. D� �� ���� ��������
����� �������� ������������?

I: M��� c�������� a�� ���� �a�������, ��� �����c� �� ���� ����ab��.


I: Y��, ��c���ca��� ���� ����� b� �����b��. Y�� ����� ���� �� �a�� ���� ��a� ��� b�� ������
ca�ac��� �� ��� I������� ������. L���� a����� ��� ����� ��� ca� ����� �� ����c��� ����������a�
�� ��� ca�ac��� ��� b�� a�� ��� c���� a���c�a��� ���� ��.

C: OK, �� �������� ��� ����� ����� �������� ��� ���� ��� ���������� �� E�� 2 ��� �����,
����� �� ����� �������� ������ � ���� ������ ������, ����� E�� 30 �� ��. I� �� �����
��������� ��� ��������, ��� �� ���� �� ��� ��������� ��������� �� ��� ������� ��������, ����
����� ��������� ������� ��� ����. F����������, I �� ����������� �������� ���������� ��������,
���� ��������� �� ���� ����� �������� �� ������ ��������. O� ������ �� ����� ���� �� ���� �
�������� �������� ���� ��� ����, ��� ����� E�� 12 ��� �� ��� 20 �� ������ �� ����� �� MDF
������, ��� ����� ������� �� ��� �����, ����� ������� ���� �����, ����� ������� ������� ���
����. I ����, �� ����� ������ ��� ������� ���� ��������� ������ �������� ��������.

I: OK, ������ ���� ����� ���a� a�� ����� ��������a����. A������� ���� ��� ca� ����� ��?

C: ���� ����� �������� �� � ������� ������ �� �������, ������� �� �������� ��� ������ �������
 ���� ��� �����? I� ��� ������ �� ��������� ��� �������� ����� ������ ��� �������?

I: G��� �����! T�� a��a �� �ac� ������ �� ���� �� ���� ��� �a��, ���c� ��a���

C: ��� ���� � ���� ��������� �� ��� ������ �� ���������� ��� ��������. I� �� ���� ��� ����
������� �����, ��� �������� ��� O��� ����, ���� ���� ����� ������ ������, �� ��� ������� �����
50% �� ��� ���������� �������, I ����� ����, ����� ���� 20% �� ��� ��������� ��� ���� ����
20% �� ��� AD�L ��������� ����. ���� ����� ���� � ����������.
10.6 C��� ������ ���� (BCG)

��� ���� ���������

T���� �� a c���a��, ���'� �a� a ��a��� a���c�, ��a� ���� �� a c����a����� ca�� c����� ����� a��
��c����� ����� ca��� ���� a�� b�a�c��� a�� �������� ��. S������ a���� ������� �� ��� ca��
c�����, ��� ��a��� a���c� �� �����a��� ����ac�ab�� b�ca��� �� ��� �a��� a����� �� ����� ca���
��c�����.

B����� a�������� a�� ���������, ��� ca����a�� ������ c��c� �� ��/��� c���������
�������a��� ��� ���b��� ��a���. T�� ca����a�� ca� a�� a�������a� ��������� �� ��� ���b���
�� ��� c��������� c��a�.

T�� ����� �������� ����� ��� ca����a��'� c��a������ a�� ab����� �� c��� �� ���� ���� ��a� ���
���a, a� ��� �a�� ���� �� ����� ��� ��� �� a ����c����, ���c� ����� c����� �� ���� ����
���a�. F����, ��� ca����a�� ������ ����� �� �a�� ca����, a�� ���� ���c��� ����� �a�� ca����
�� ���� ���a���� �����b�������.

I����������: W�a� c���� ca��� ��� ������a� �� ��� ca�� c�����?

C��������: I ������� ����� ����� �� ��������� ����� �� �������:

C����� �� ������
A �����b�� a����� c���� b� a� ����� �� ��� ���� ������:

N��b�� �� ��������
c��������

N��b�� �� ca��� ���
Ca�� ������
c�������

� S�a���a� �a����� (%)

R������� ���b�� ��
a����� Ca�� ���� (��c)

A���a�� ���������� ���� +


W�a���� ���� (��c)

: W������ ����� ��� a����


(��c)

Ca�ac��� ��� a���� G����/��� �a��� (%)

W������ T��� D��a����
(%)
M��� ca�� ���������� �a�� ���� c�����a���� �� ����. T� ����� ��, ��� ca����a�� ���� �a�� ��
������ ��� ��� ���b��� ��� a�� �a��� �� ��� "�����������" �� c������ ��� �������� ���b�� ��
a�����. T���� a�� �����a� �a�� �� ������ ��� ��� ���b���. F���� �� a��, a� ����������� �����
���� ��� ca����a�� ���� �a�a �� ��a�� ����, a�� ��/��� ���� �a�� �� ��c� ��� ��� ���b��� ��
��� �� ��� ca�c��a����. S�c�����, ��� ca����a�� ca� a�� ��� ����������� ��� �a�a, a�� �������
��� ca����a�� ca� �a�� a���������� ����� c����� �����. T�� ca����a�� ������ a��a��
c��a��� ��a�� ���� ��/��� �� �a���� a� a��������� a�� ��� ��/��� c���� �� ���� ��.
I� ���� ca�� ���������, ��� ��������� �a�a �� ����� b� ��� �����������:

Cumulative number of customers who are


diverted to the ce ntralized call center
Number of 1.200.000
customers

1.000.000

800.000

600.000

400.000

200.000

0
0 10 20 30 40 50 60
F��� ���� ��a��, ��� ca����a�� ca� ����c� ��� c����a���� ���b�� �� �������� c�������� ��
���� 20: a�������a���� 140.000 c��������. S� �� ���� 20, ����� a�� 140.000 c������ "�� ���
�a�aba�� �� ��� ca�� c�����". T� ca�c��a�� ��� �������� ���b�� �� a�����, ����� �a�a �� ���
����������� �� ������, a�� ��� ca����a�� ���� �a�� �� ������ �� ���. H�/��� ca� ��a�� ��� ����
a����� ��� ����������� ��� �a�a, b�� ��'� a��a�� b��� �� �a�� a���������� a�� ���� �����
���� ��� �����������. F�� ��a����: "I a����� ��a� �� a���a��, a c����� �a� c���ac� ���� a�
a���� �� ��� ca�� c����� ���c� ��� ��a�, �� ��� a���a�� ���b�� �� ca��� ��� c������� �� 2".
A��: "I ������ �� ����� a�� �a�� ���� ca��� �� ���� 20 ������ ��� ����� ����� �� ��� ��a�,
��� ��� I a����� ��� ��a���a� �a����� �� b� 0%". T�� ca����a�� ���� ca�c��a��� ��� ca���'
������: 140.000 / 52 * 2 + 0% = a�������a���� 5.400 ca��� �� ���� 20.

T� ��������� ��� a���a�� ���������� ����, ��� ca����a�� ca� a�� ��� ����������� ���
������a����. T�� ����������� �a� ������ ���� ��� ca����a�� �� �a�� a� a���������, �� ����
��� ���b���. I� ���� ca��, ����� a�� �����: "M��������� �a��� a�� ��a��������� �a�� a�
a���a�� ca�� ���� �� 240 ��c���� a�� a� a���a�� ��a��������� �� 60 ��c���� ��� ca��". T���
������a���� ����� ��� ca����a�� ��� a���a�� ���������� ����: 240 + 60 ��c���� = 5 �������
= 1/12 ����.

T�� �a�� ������ �� ��� ca�ac��� ��� a����. T�� ca����a�� ca� a����� ��a� ��� ���b�� ��
����� a� a���� ����� �� 40 ��� ����. T� ��������� ��� �����/��� �a���, ��� ca����a�� c����
a�� ��� �����������, ������ ��/��� c���� a��� �a���� �a�� a� a��������� �� ���
��������c�: "I a����� ��a� �� ��� 8 ����� ������ �� a �a�, a�������a���� 1.5 ���� �� ���� ��
���c� a�� ����� b��a��. T��� �a��� ��� �����/��� �a��� �80% ((8�1.5)/8 = 81%). T�� �a��
T��� �������� ���� ���� �� ��� ca����a�� ��a��� ��a��� ��� ���b��� a�� �� ��/��� ���� ���/���
����c���� �� ��� ����� �a�. L������ a�a�� a� ��� ����c����, ��� a����� �� c��a�: T���� a��
����� ������ �� ��c��a�� ��� a����� �� a����� ������:

1. R���c� ��� ca�� ������


2. R���c� ��� a���a�� ���������� ����
3. I�c��a�� ��� ca�ac��� ��� a����

T� ��������� ���c���c ���������, ��� ca����a�� ����� c��a������ a�� �������a����� �� ���
���b���.

O� ��� ����� �����:


S��� ��������� ��� ca��� ���� b�a�c��� �� ca�� c����� (a�� ������ bac�)
I����� a c��������ba��� ���� ���c� a������ ��� ���� ���������� a���� ���������

A �����b�� �������� �� ��� ��c��� ����� ��:


R���c� ��� ��a���� ���� b� ���������� ��, ��������� ��� ���� c����� ��a����
ac����� a�� ������� ����� ac����� ����� �����b��. F�� ��a���� �������c���� �� a
c������� a����ca���� ��a� ����� ��� a����� ���� ��� ��a��a�� ��a���� ac����� ��c�
a� "������� a b��c����"

F�� ��� ����� �����, a �����b�� �������� ����� b�:


A���� ��a����� �� a����� ���� ac��a� ���a�� ��� a����� b� ��b������� ���� a�����
This Practice Case was sponsored by:

The major cost driver for the jet appears to be purchased materials. Within manufacturing, direct labor is a fairly large component
of cost, as are program management and corporate overhead within overhead. I think we would want to concentrate most on
materials, however, since that's where most of the costs can be found.

 That sounds like a good place to start. Where would you look within materials?

I see that materials are broken down into purchased subassemblies, components, and raw materials. I understand what raw
materials would be, but what would be the difference between components and subassemblies?

 A subassembly functions on its own. An example is the pilot night vision system. A component is a smaller
This Practice Case was sponsored by:

But remember, if your client loses the contract, it will lose its customer unless it is teamed with the competing
bidder. Even then, if the competitor is underbidding your client, there will be even less room for it to profit.

Perhaps it would have an incentive to reduce its costs in order to maintain the contract. Are the majority of its costs in materials as
well?

How could you find that out?

I would want to interview the purchasing and engineering personnel of the different subcontractors in order to understand their cost
structures. If we had a better understanding of their economics, our client might be able to reduce cost across the board, allowing it
to compete more effectively for the contract without killing everyone's margins.

Let's say that purchased materials average approximately 70 percent of the price paid to most of the
manufacturers.

If the cost of subassemblies represents 40 percent of the jet cost and 70 percent of that is purchased materials, total purchased
materials would be approximately 28 percent of the cost for subassemblies. Purchases of raw materials and components represent
another 15 percent, for a total of around 43 percent of the cost of the jet. If our client could reduce the cost of raw materials by 20
 percent, it could reduce the cost of the jet by more than 8 percent, more than enough to offset the 5 percent reduction it would need
to win the contract.

 That sounds reasonable, but 20 percent is a very lofty goal. How would you go about doing that?

First, I would look at the number of suppliers. Are there a large number of suppliers to the subassembly manufacturers?

 The client estimates that there are approximately 125 suppliers of raw materials and components among the
manufacturers of the subassemblies and itself.

Well, that sounds like a large number of suppliers. Of course, they could be providing very specialized materials to the subassembly
manufacturers. Are these suppliers providing customized or more commodity products?
This Practice Case was sponsored by:

S TEP 5: SUMMARIZE AND MAKE RECOMMENDATIONS

Our client needs to reduce costs by 5 percent. The largest area of opportunity appears to be in purchased
materials, the majority of which comprise subassemblies manufactured by seven subcontractors. By looking at
its purchases in total, the client can target approximately 40 percent of costs.

 To achieve the 5 percent cost reduction, it would need to reduce costs by 15 to 20 percent. It could try to do
that by increasing commonality in the design of the subassemblies and components and by shifting volume to
a smaller number of suppliers.

Considering that the majority of the raw materials and components are purchased commodities, do you think
the 15-20 percent cost reduction is achievable?

Well, I know that typically have lower margins than more customized products. I suspect it may be challenging to hit the client's
savings target by focusing only on these purchases. But since raw materials and components represent about 40 percent of costs and
there is an opportunity to concentrate purchasing, I think we should start here.

 Where else could you look for savings?

If I look back at the cost data on the jet, direct labor is another large cost component. As a contingency, we could look into that
area as well. I've read that other companies use outsourcing to lower their manufacturing costs-perhaps our client could do the same.
For example, it might want to increase its use of purchased subassemblies and reduce the amount of direct manufacturing it does.
Of course this would work only if it could drive direct labor costs below the offsetting cost of these subassemblies. The client will be
working closely with the subassembly suppliers to implement its purchasing initiative. This may give it an opportunity to explore
the suppliers' capabilities at the same time.

 That's an interesting suggestion. How would you recommend the company pursue both of the initiatives you
have discussed?

I would look first to combine purchases across the subassembly suppliers with our client's purchases. I suspect that the client and
the subassembly suppliers will need to share a great deal of information, including engineering drawings and specifications, with
This Practice Case was sponsored by:

Discount retailer Case

S TEP 1: A CTIVELY LISTEN TO THE CASE

 Your client is the largest discount retailer in Canada, with 500 stores spread throughout the country. Let's call
it CanadaCo. For several years running, CanadaCo has surpassed the second-largest Canadian retailer (300
stores) in both relative market share and profitability. However, the largest discount retailer in the United
States, USCo, has just bought out CanadaCo's competition and is planning to convert all 300 stores to USCo
stores. The CEO of CanadaCo is quite perturbed by this turn of events, and asks you the following questions:
Should I be worried? How should I react? How would you advise the CEO?

S TEP 2: ESTABLISH AN UNDERSTANDING OF THE CASE

So, the client, CanadaCo, is facing competition in Canada from a U.S. competitor. Our task is to evaluate the extent of the
threat and advise the client on a strategy. Before I can advise the CEO I need some more information about the situation. First of
all, I'm not sure I understand what a discount retailer is!

 A discount retailer sells a large variety of consumer goods at discounted prices, generally carrying everything
from housewares and appliances to clothing. Kmart, Woolworth, and Wal-Mart are prime examples in the
U.S.

S TEP 3: SET UP THE FRAMEWORK 

Oh, I see. Then I think it makes sense to structure the problem this way: First, let's understand the competition in the Canadian
market and how CanadaCo has become the market leader. Then let's look at the U.S. to understand how USCo has achieved its
 position. At the end, we can merge the two discussions to understand whether USCo's strength in the U.S. is transferable to the
Canadian market.

 That sounds fine. Let's start, then, with the Canadian discount retail market. What would you like to know?

S TEP 4: E VALUATE THE  THE


This Practice Case was sponsored by:

CanadaCo's cost structure isn't any lower than the competition's. Its higher per-store profits are due to higher
per-store sales.

Is that because it has bigger stores?

No. CanadaCo's average store size is approximately the same as that of the competition.

If they're selling similar products at similar prices in similarly-sized stores in similar locations, why are CanadaCo's per-store sale s
higher than the competition's?

It's your job to figure that out!

Is CanadaCo better managed than the competition?

I don't know that CanadaCo as a company is necessarily better managed, but I can tell you that its
management model for individual stores is significantly different.

How so?

 The competitor's stores are centrally owned by the company, while CanadaCo uses a franchise model in which
each individual store is owned and managed by a franchisee who has invested in the store and retains part of
the profit.

In that case, I would guess that the CanadaCo stores are probably better managed, since the individual storeowners have a greater
incentive to maximize profit.

 You are exactly right. It turns out that CanadaCo's higher sales are due primarily to a significantly higher level
of customer service. The stores are cleaner, more attractive, better stocked, and so on. The company
discovered this through a series of customer surveys last year. I think you've sufficiently covered the Canadian
market-let's move now to a discussion of the U.S. market.
This Practice Case was sponsored by:

So it seems that USCo has been so successful primarily because it has lower prices than its competitors.

 That's partly right. Its success probably also has something to do with a larger selection of products, given the
larger average store size.

How did USCo get so much bigger than the competition?

It started by building superstores in rural markets served mainly by mom-and-pop stores and small discount
retailers. USCo bet that people would be willing to buy from it, and it was right. As it grew and developed
more clout with suppliers, it began to buy out other discount retailers and convert their stores to the USCo
format.

So whenever USCo buys out a competing store, it also physically expands it?

Not necessarily. Sometimes it does, but when I said it converts it to the USCo format, I meant that it carries
the same brands at prices that are on average ten percent lower than the competition's.

What criteria does USCo use in deciding whether it should physically expand a store it's just bought out?

It depends on a lot of factors, such as the size of the existing store, local market competition, local real estate
costs, and so on, but I don't think we need to go into that here.

Well, I thought it might be relevant in terms of predicting what it will do with the 300 stores that it bought in Canada.

Let's just assume that it doesn't plan to expand the Canadian stores beyond their current size.

OK. I think I've learned enough about USCo. I'd like to ask a few questions about USCo's ability to succeed in the Canadian
market. Does USCo have a strong brand name in Canada?

No. Although members of the Canadian business community are certainly familiar with the company because
of its U.S. success, the Canadian consumer is basically unaware of USCo's existence.
This Practice Case was sponsored by:

 That's probably true, but remember, CanadaCo must also cope with the same high labor costs. Can you think
of additional costs incurred by USCo's Canadian operations that would not be incurred by CanadaCo?

USCo might incur higher distribution costs than CanadaCo because it will have to ship product from its U.S. warehouses up to
Canada.

 You are partially right. CanadaCo has the advantage in distribution costs, since its network spans less
geographic area and it gets more products from Canadian suppliers. However, since CanadaCo continues to
get a good deal of product from the U.S., the actual advantage to CanadaCo is not great-only about two
percent of overall costs.

 All this suggests that USCo will be able to retain a significant price advantage over CanadaCo's stores: if not ten percent, then at
least seven to eight percent.

I would agree with that conclusion.

S TEP 5: SUMMARIZE AND MAKE RECOMMENDATIONS

I would tell the CEO the following: In the near term, you might be safe. Your stores have a much stronger brand name in
Canada than USCo's, and they seem to be well managed. However, as consumers get used to seeing prices that are consistently
seven to eight percent less at USCo, they will realize that shopping at USCo means significant savings over the course of the year.
 Although some consumers will remain loyal out of habit or because of your high level of service, it is reasonable to expect the
discount shopper to shop where prices are lowest. Moreover, over time your brand-name advantage will erode as USCo becomes
more familiar to Canadian consumers. You certainly have to worry about losing significant share to USCo stores in the long term.
You should probably do something about it now, before it's too late.

Can you suggest possible strategies for CanadaCo?

 Maybe it can find ways to cut costs and make the organization more efficient, so it can keep prices low even if its cost of goods is
higher.

 Anything else?
This Practice Case was sponsored by:

Medical Software industry Case 

S TEP 1: A CTIVELY LISTEN TO THE CASE

 Your client is GenCo, a large, international, diversified company with a health care division that produces a
 wide variety of medical instruments and related services. Five years ago, it expanded into the health care
software industry by purchasing MedCount, which markets administrative systems to large U.S. hospitals.
 These systems are designed primarily for back-office functions; they are not designed for managing patients or
providing other physician and technical support. Since it was purchased, the software division has failed to
deliver the growth needed to justify the multiple GenCo paid for it. GenCo feels it has already squeezed
margins as much as possible, and now is looking for new sales opportunities. MedCount turned to BCG to
help identify potential ways to increase revenues. How would you approach this problem?

S TEP 2: ESTABLISH YOUR UNDERSTANDING OF THE CASE

First, let me make sure I understand the problem. The parent company produces medical devices and services, but before the
acquisition was not involved in health care software. The company it purchased, MedCount, sells only administrative systems
software to large hospitals. It is now looking for opportunities to increase re venues.

 That is correct.

Could I take a moment to jot down a few thoughts?

Sure, that would be fine.

S TEP 3: SET UP THE FRAMEWORK 

I would suggest using the following framework: First, I'd want to understand the market size and growth rates for MedCount's
market and related software markets. Next, I would like to explore the competition and their market shares. Third, I would like
to examine customer requirements and then, given those external conditions, look at the division's capabilities to understand how
well prepared it is to meet the needs of the marketplace.
This Practice Case was sponsored by:

 Administration Patient administration Physician support


Market size ($M) 1,500 1,000 1,200
Growth rate 5% 5% 12%

From a size and growth perspective, physician support systems looks like a very attractive market. I'd like to know a little about
the customers themselves. The client is currently targeting large hospitals. Approximately what percentage of the market do they
represent?

 We were unable to get an exact breakdown, but we know that these hospitals make up the vast majority of the
total medical software market.

That would make sense, since the more sophisticated procedures at a hospital might necessitate more advanced software solutions. I
know that there have been a lot of changes in the industry as a result of managed care. I don't know much about the industry, so I
would want to look at market studies and press clippings to get a better sense of the hospital market in general and any technology
or software trends more specifically.

Okay. Let's say that you did that and were presented with this summary of market trends:

• Consolidation in the industry, with three to four large hospital networks dominating 45 percent of the
market
• Cost controls instituted, particularly as these large hospital networks acquire smaller hospitals
(centralization of functions being a key cost issue)
• Many hospitals seeking to consolidate their vendor base
•  With regard to technology, many hospitals upgrading their older systems

If hospitals are consolidating vendors, perhaps our client has an advantage in being part of a larger medical company. Maybe the
client could also gain some advantages by expanding into other software segments. Are the people responsible for purchasing
software at the hospital the same for all three segments?
This Practice Case was sponsored by:

Patient Administration Sales ($M) Growth (%)


HTI 300 5%
Registration Software Solutions 240 4%
Signup Software 60 3%
HCS Software Systems 30 16%
Patient Software 20 -1%

Physician Support Sales ($M) Growth (%)


HCS Software Systems 150 16%
Physician Support Systems 100 11%
Medical Technology Inc 25 18%
HTI 20 32%
MedSys 5 15%

Very interesting. The first thing I would note from the data is that the market concentrations are very different. In administrative
systems, the top five competitors control 66 percent of the market and in patient administration, they control 65 percent. But in the
 physician support market, they control only 25 percent.

I would want to know what gross margins look like in each of these markets as well. I might turn to analyst reports and look at
competitors' financial statements to deduce whether they are making money in each market

Gross margins vary, of course, but the analyst reports have margins of 25 to 30 percent for administrative
systems and for patient administration. For physician support, the margins tend to be higher, more like 45 to
50 percent.
This Practice Case was sponsored by:

important for MedCount to have a product offering in each of the three market segments. That should not be too difficult, since the
company is already in the software industry.

Perhaps, but you should think a little more closely about these types of software. Are all software systems
alike?

Well, let me think about that for a moment. I suspect patient administration would have relatively low entry barriers. From your
earlier description, these systems appear to be pretty basic, dealing primarily with admissions and patient tracking However, the
entry barriers in physician support might be higher, since these systems are more complex and there are probably multiple systems
 for the various physician procedures. I guess it would be harder to get into those types of systems.

 That would make sense.

Since the company might want to go into only some of the segments, I would want to know how important it is to have products in
all three segments. Do we know if the competitors are marketing their products as a bundle?

How might you find that out?

Since it would be difficult to talk to a competitor directly, I would probably target a competitor's customer, particularly one that
 just converted from our client's software.

Let's say you get an interview with a customer that recently switched to HTI. You discover that the
competitor was offering it a better pricing deal and service for software products in all three segments.

How were MedCount's software and service perceived in relation to those of competitors?

 The customer thought that its administrative systems were adequate, "the old standby," but not stellar.

Were there any other key reasons it switched from MedCount's system?
9.7 Supermarket deli turnaround case (BCG)

Questions and Facts

1. Client’s deli financials

See Exhibit 1

2. Overall industry/ customers

Deli meat category has been flat to slightly declining recently. Prepared foods category has
been growing at roughly 10% per year as people have less time to cook at home.

3. Competitors

Increasing competition from deli departments of other supermarkets, discounters, etc. – e.g.,
expanding product lines, increasing advertising. Also competes with fast food restaurants in
prepared foods category.

4. Client’s product mix and recent events

Mix has remained constant, with the exception of two products introduced a couple of years
ago – BBQ chicken wings and “made to order” sandwiches. Both products have been a major
boost to prepared foods revenue.

5. Info on new products


SUPERMARKET DELI TURNAROUND
Exhibit 1

$M
2002 2003 2004
Deli meats Revenues 260 255 260

COGS 160 155 160

Prepared
Prepared foo ds Revenues 360 400 440

COGS 190 230 270

Overall Revenues 620 655 700

COGS 350 385 430

Gross margin 270 270 270

SUPERMARKET DELI TURNAROUND


Exhibit 2
SUPERMARKET DELI TURNAROUND
Sample Approach

Main question What should supermarket


supermarket do to turn aroun d deli?

Key areas to Reven ue


ue an
an d p ro
ro fifi t br
break do
do wn
wn wi
wi th
th in
in de
del i Ex te
tern al
al fa
fac to
to rs
rs in
in flfl ue
uen ci
ci ng
ng t he
he ov
ov er
er al
al l d el
el i
explore market

• Deli meat
meat revenue and profits flat - consistent • People have less time to cook at home –
with overall category prepared foods category growing, deli meats
• Prepared
Prepared foods showing revenue growth (10% category flat
consistent with category) but no profit growth • Increasing
Increasing competition from other deli
 Anal ysi s  – ther efo re decl ini ng m argi ns – why ? departments – starting to expand product lines,
- made-to-order (MTO) sandwiches losses increase advertising, etc.
offsetting profit growth from BBQ wings

• Eliminate made-to-order
made-to-order sandwiches (at
(at least in l ow-traffic stores or during non-peak hours)
Recommend- • Raise or lower prices on MTO sandwiches (depending
(depending on demand elasticity)
ation • Boost demand for MTOs andwiches (e.g.,
(e.g., advertising,
advertising, promotio ns, merchandising)

• Eliminating MTOs andwiches or boosting demand can impact overall traffic in store and
and deli
Other factors

Framework and Analysis

 There are three main questions asked to the candidate:


•  Which part of the business is responsible for the lack of profit growth – deli meats,
prepared foods, or both?
By doing a back-of-the-envelope analysis of product profitability (based on data in Exhibit 2),
the candidate can find that BBQ wings have a 50% margin, indicating that they are not a
problem. On the other hand, he/she will find that the client is losing a lot of money on the
“made to order” sandwich concept.

 The candidate is then asked for recommendations, which could i nclude:

1) Eliminating the “made to order” sandwich


2) Restricting the “made to order” sandwich to busier stores or during busier times of the day
(e.g., lunch hours only)
3) Raising or lowering prices (to either increase profit per sale or units sold – will depend on
demand elasticity)
4) Boost demand (through increased advertising, promotions, better merchandising, etc.).

 The candidate can also consider the second-order effects of eliminating the product or
boosting sales (the effect on traffic in the deli and the overall store).

9.8 China outsourcing case (BCG)

Problem set-up

 The client is a national manufacturer of plastic consumer products that are sold in a variety of
retail formats, including supermarkets, discounters, club stores, and dollar stores. The company
has three main product lines: 1) freezer bags, 2) plastic plates and utensils, and 3) specialty
plates and utensils.
2. Plastic plates and utensils

Disposable plates and utensils; intended for single/limited use. According to customers, top
purchase criterion is price. Client is #2 in category, with 300 million lbs. sold. Client is at cost
parity with category leader but has a weaker brand.

3. Specialty plates and utensils

Plastic plates and utensils produced for specific retailers, customized to their design specs.
 According to customers, top purchase criterion is style/design. Because many products are
new and untested, demand is highly variable. Client is #1 in category, with 100 million lbs.
sold. No strong competitors.

4. Current client production capabilities

 All products are made in


i n a single factory in Ohio. The factory is at capacity and the company is
considering building or acquiring a nearby facility.

5. Chinese production options

Client has no previous experience in building and managing a factory overseas. Client has met
 with several Chinese manufacturing partners and has done initial pr oduct testing.

 All three product lines have similar cost structures


structur es and savings (Exhibit 1).

Quality: lower quality on freezer bags, equal quality on plastic plates and utensils (both regular
and specialty).
CHINA OUTSOURCING OPPORTUNITY
Exhibit 1

Costs in Costs in China Costs in


Costs U.S. ($/lb.) relative to U.S. China ($/lb.)
Labor  0.30 8% of wage rate
80% of productivity
Material
• Plastic resin 0.30 80%
• Other material (incl. packaging) 0.20 75%

Variable overhead 0.05 140%

Fixed overhead 0.10 60%

Transportation
• China to U.S. distribution center  N/A $6K to ship 40K lbs.
• U.S.
U.S. distribution c enter to customer  0.05 Same

Total 1.00

CHINA OUTSOURCING OPPORTUNITY


Sample Approach
Framework and Analysis

 The candidate should start with a brief overview of the potential benefits and risks of
outsourcing to China. The main benefit is lower costs, mostly driven by inexpensive labor. A
secondary benefit is a possible springboard into the emerging Chinese (and other Asian)
market. Risks include lower labor productivity, possible quality issues, longer lead times,
additional transportation costs, and potential communication/coordination issues. The
candidate can be asked about the ramifications of longer lead times – they include greater
carrying costs, higher cycle and safety stock, greater forecast error, and less responsiveness to
demand.

 There are three main questions asked to the candidate:


• How much cheaper is producing in China?
•  What do consumers value and how would outsourcing affect those criteria?
•  What are the client’s current production capabilities and how would outsourcing
part/all of their production affect the remainder?

First, the candidate should size the opportunity – is this a $5 million or a $500 million
opportunity? By solving for the last column in Exhibit 1, the candidate will find that the client
 would save $0.25/lb. (25% of current costs) by outsourcing to China. Given current
production levels, the client would save $50 million by outsourcing freezer bags, $75 million by
outsourcing plastic plates and utensils, and $25 million by outsourcing specialty plates and
utensils. Two notes: 1) costs may increase if the Chinese Yuan rises versus the dollar and 2)
these estimates do not include a profit margin for the Chinese outsourcing partner.

 The candidate should also recognize that cost savings alone are not sufficient to make a
decision. It is important to understand how an outsourced product will affect sales. The
 An analysis of customer
cu stomer purchase behavior indicates that plastic
pl astic plates
p lates and utensils should be
outsourced, specialty plates and utensils should not be, and freezer bags probably should not
be. The final step is to understand the impact of outsourcing on the client’s current production
capabilities. For example, will it lead to plant closings (resulting in closing costs and possible
negative publicity)? Will it lead to underutilization of current facilities?

Since the current plant is already near capacity, moving plastic plates and utensils offshore
 would actually save the client from investing in new facilities. However, since that product line
makes up 50% of total production (in terms of lbs.), removing it may create too much extra
capacity in the current plant for the two remaining lines. To compensate, the client could
produce a new product line, rent out the extra capacity, or move to a smaller plant.

9.9 Specialty paper sales case (BCG)

Problem set-up

 Your client is a leading manufacturer of specialty papers sold to commercial printers. The
client produces self-adhesive sheeted papers that are ultimately used in a variety of labeling
applications – including the labeling of consumer goods and the printing of self-adhesive signs.

 Your client’s operations are profitable, but the business has failed to grow over the past few
years. The client would like to invest in the business and you have been asked to identify
opportunities for growth.
GROWING SPECIALTY PAPER SALES
Questions and Facts (II)

 All fig ur es ar e per eq ui valen t bo x


Small (boxes) Medium (carton) Large (pallets)

Price to printer 20.00 18.00 15.00

Materials 5.50 5.50 5.50

Coating 1.00 1.00 1.00

Sheeting 0.50 0.50 0.50

Packaging (direct costs) 3.00 2.00 1.00

Gross prof it/($ per equivalent box) 10.00 9.00 7.00

Number of printers 20,000 3,000 1,000

 Ann ual u sag e (num ber of eq ui valen t 100 500 3,000


boxes)

Total potential profit pool ($) 20,000,000 13,500,000 21,000,000

Framework and Analysis

 The problem set-up indicates that the client wants to invest in this business. Investment can
take many forms including expansion of manufacturing operations and capacity, expansion of
customer-facing activities, and acquisition of competitors. It is interesting to note that this is
 The candidate should recognize that a comprehensive solution evaluates the required
investment to serve a particular market segment (packaging line, manufacturing operations,
additional SG&A) against the expected return.

 The candidate should evaluate the profit pool from serving medium and large customers.
cu stomers. This
should be based upon an assumption about the size of the market that the client can capture.
 Assuming the client can match its small printer market share, the client could capture an
additional 20% of the medium or the large printer customer segment

 The following is given to the candidate:


• Investment and operation of carton packaging line would cost $675,000 per year
• Investment and operation of the palletizing line would cost $2,300,000 per year

 A logical conclusion would be that an investment in a carton packaging line would be a


superior investment compared to the palletizing line

Potential conclusions:
•  The carton packaging line is a less-risky investment (requires less up-front capital)
•  The solution assumes a static environment. If large printers
pr inters are growing in number and
or usage of specialty paper, this may change the answer
•  The investment in a new carton packaging line would need to be evaluated against
other potential investments to understand if it is the optimal use of the client’s capital

9.10 Oil tanker case (Booz Allen Hamilton)

Step 1: Background and question


Case 13: Street Sweepers

BACKGROUND
Firm: BCG
Round: 2006 FT, Final
Content: Mainly qualitative

CASE QUESTION

Our client is a large industrial conglomerate that has $60-70 million in annual revenues and is
extremely profitable. They have engaged BCG to examine one of their business units that is
underperforming. The unit manufactures and distributes street sweepers (zamboni/lawnmower-
like machines that a person would ride and which uses two large rotating brushes to sweep up the
street as it moves by) and has been doing so for 20 years. While never a standout division, until
recently it had always been profitable. 4-5 years ago the profit margins started to fall and it is
currently just above breakeven.

Management of the conglomerate had made a decision 2 years ago that they did not want to
invest in new features for their product lines and has the approach that they don’t want to invest
significant amounts of capital now (unless they can be convinced otherwise).

BCG’s task is to analyze the unit’s performance and recommend to the management of the
conglomerate what should be done. Tell me some of the areas you’d consider looking into here
knowing that we only have a few weeks to finish this engagement.
Interviewee: It might be happening because of trends in the market, changes in customer
 preferences…

 Interviewer: Since you mention it, who are the customers?

Interviewee: I suppose they would be municipalities, governments, etc. I don’t see this as
 being something for individual use.

 Interviewer: Usually not. That’s right, it’s mostly state and local governments. So we looked
into the customers and they are keeping with past trends, replacing their street sweepers every 4-
7 years, and the number of municipalities purchasing street sweepers has remained constant.
What else might be driving revenues down?

Interviewee:  Prices could be coming down…

 Interviewer:  Prices have actually remained constant.

Interviewee:  So how about competition? Have new competitors entered the market or stolen
market share?

 Interviewer: There have been no new competitors. These are what sales have looked like 20
 years ago to now. What do you want to know after looking at this?

100%
 Interviewer: About 5 years ago, B introduced a new technology that used air vacuums to clean
 streets instead of the mechanical collecting methods that had always been used in the past.
These air machines are more effective at picking up small debris like sand and small litter, and
work more quickly and efficiently than the mechanical ones previously offered. Price points are
about the same. B still sells mechanical machines, but the drop in our market share was directly
related to the new air offering.

Interviewee: So how about C?

 Interviewer: C makes machines that are far more heavy-duty, it’s really a different type of
offering.

Interviewee: So we don’t compete with C in reality?

 Interviewer: No, but we could. Do you think we’d want to?

Interviewee: Not likely without knowing much about the market. They seem to have been
stable and have an expertise, so unless we can offer something new to their customers I’d guess
they’ll defend their position in a niche market at all costs.

 Interviewer: Good, that’s the conclusion we quickly came to on C. Now, what other
information do you need to recommend something for our client?

Interviewee: I need to know if we can replicate the air technology.

 Interviewer: We can, but it will take 2 years and cost $25 million.
Interviewee: I’d think they’d need mechanical ones to deal with larger, more solid things on
their streets. It looks like in the south they can use more air machines which would make sense
 b/c they’d have sand and small debris. In the north you might get larger rocks, chunks of ice,
etc. This tells me the market isn’t going to zero, so maybe the company can simply protect the
market share that it has, scale back production to the point where the market will be in
equilibrium between air and mechanical sweepers, and keep decent margins going forward.

 Interviewer: Right, so that’s exactly what we recommended. Once we recognized these trends,
we also looked for a strategic buyer, and in fact the conglomerate sold the unit to a foreign
company that already had air technology developed. That was outside the scope of this case,
though – just an interesting follow-up.
Case 14: Eye Surgery

OVERVIEW
Firm: BCG
Round: 2006 FT, First
Content: Market sizing, qualitative and quantitative

CASE QUESTION

Our client is a manufacturer of equipment for eye surgery. Specifically, the machines measure
deficiency in eyes, and the company also produces lasers for post-operational procedures and
adjustments. They don’t actually make the lasers or devices used for Lasik – rather, they are
complementary products for this procedure.

The global market for these devices is growing, but at a declining rate. As a result, the client
wants to get into a higher growth area, so they are looking at acquiring a company that makes
inter-ocular devices. These devices are used instead of Lasik but with similar effectiveness, and
they are used for two major categories of patients:

1 – patients with cataracts


2 – refractive surgery (to correct near or far sightedness)

How would you approach this opportunity? What would you look at?
DRYWALL COMPETITIVE THREAT

Question 2: Now ask the candidate if our client maintains the price how
much volume it potentially needs to lose to make it equivalent to
cutting the price by 10%.

If the candidate takes the initiative and explores this question then it needs
to be noted and encouraged (this differentiates a very strong candidate
from others)

Expected Analysis:  Let the new volume of units the client sells be “x”, and if
the client sells “x” units @$10:
• Revenue = 10*x Variable Cost = 3*x Fixed Cost = $200,000
• Profit = 10x -3x - $200,000
 A classic mistake is to take the contribution per unit as $5 (effectively
treating the allocated fixed cost as a variable cost too) and using $5*x as
the profit.
DRYWALL COMPETITIVE THREAT

So for this option to be equivalent to cutting the price by 10% the


equation would be
• 10x –  3x –  200,000 = 400,000
• 7x = 600,000
• x = 600,000/7 = 85,700 (approx)
Loss in market share = (85,700-100,000)/100,000 = 14.3%

It is expected that the candidate explains what this 14.3% number


 represents; expected answer
would be that the 14.3% is the maximum market share our client can
afford to lose before the option of cutting price by 10% appears to be
a better choice.

 A candidate can differentiate himself or herself by stating that the key


is to estimate how much the client can potentially lose if it does not
 reduce price by 10%.
DRYWALL COMPETITIVE THREAT

Question 3: Now ask the candidate that if our client needs to make a
choice between these two options how you would help our client make
the choice.

Now the candidate is expected to make use of his or her plan to help make a
decision. If candidate asks the following questions provide the following
information:
• What is the market structure? (In terms of number of competitors and their share)
• Show Exhibit 1, with the shares
• What is size of the competitor?
• They are a small to mid size player
• Who are our customers?
• They are primarily contractors/small business owners. They buy directly from us.
• What are our customer’s preferences or how price sensitive they are?
• Customers usually buy based on relationships and they build preference on a
particular brand. We do not have any information regarding price sensitivity.
DRYWALL COMPETITIVE THREAT

• How do we sell to our customers?


• We sell based on their orders. There are no long term contracts.
•  What are the strengths of the new competitor?
• Not much information, they are a new player in the market. They have
presence in other market our client does not compete in.
•  What are the weaknesses of the new competitor?
• Not much information, they are a new player in the market. They have
presence in other market our client does not compete in.
• What is our brand strength?
• As a number 3 player, we have strong brand presence and we have
been in the market for a significant period of time.
• How long we have been in the market?
• We have been in the market for a significant period of time.
• Is there much differentiation in the product?
• Not much, for practical purposes consider all products to be similar.
DRYWALL COMPETITIVE THREAT

Key Expected Insights


Based on the information provided the candidate is expected to
come up with at least the following insights:
• Dry wall market is a very fragmented, and we are a dominant
player

• Considering the size of the competitor, they would be one of the


small 30+1 players

• That would mean on an average the new competitors’ market


share would be around:
• 70/31 = 2.3% approx
• New Competitor assuming fair share draw would take 2.3% of our
volume, although one can argue that they may not take any
volume and compete with the other small/mid size 30 players
DRYWALL COMPETITIVE THREAT

Recommendation
The recommendation should include the following:
• The answer: Client should not reduce price, because this is not a serious
threat and cutting price in this market would lead to a price war where
everyone would lose.
• The number(s): Reducing the price by 10% would impact profit by 20%.
20% impact on profit is equivalent to losing 14.3% market share at the
current price levels and at most the client can lose 2.5% volume.
• Risks or considerations:
•We might underestimate the size or strength of the new competitor
(new competitor might be bought by a bigger player)
•One of the major competitors (A or B) may have a knee jerk reaction
and reduce price
• Next steps:
•To mitigate some of the risks our client should aggressively strengthen
its client base by providing additional services
•To create a barrier for the new competitor to poach our client’s
customers
•Client can enter into long term contracts with its customers
•Even if A or B reduces its price, our client should maintain price and
DRYWALL COMPETITIVE THREAT

Exhibit 1
CASE 6:
HOSPITAL PROFITABILITY
Firm Style Interview Round
BCG 1

Case Question:
A hospital group has been experiencing growing pains and has asked our firm for help.

Clarifying Questions & Answers


Provide the following answers only if the interviewee asks the corresponding questions.

Question Answer

What do you mean by growing pains? The hospital group originally consisted of one
hospital and then it acquired 3 other hospitals,
and its profits are going down.
Any other change in organization? Yes the CEO of the main hospital changed; it
was originally an MD doctor now it is a business
experience MBA.
Any other changes or events impacting the None.
hospital group?
Any external market condition changing? No.
HOSPITAL PROFITABILITY

Framework / Structure
This is a profitability case with lots of twists and turns. The analysis may include,
but is not limited to, the following areas:
Phase 1
• Identify the root cause of profitability decline.
•If the candidate makes an assumption that the acquisition led to
profitability decline, steer candidate away by stating that the
acquired hospital did not experience any decline in profits.
•Key expected response/insight: So the main hospital is
experiencing profit decline
• Identify the root cause of the profitability decline in the main hospital
•Costs –  No change
•Revenue  –  Decline
•Key expected response/insight: So the primary reason for profits to
decline is that the revenue for the main hospital is going down.
HOSPITAL PROFITABILITY

Explore the revenue decline


• If the interviewee asks the revenue stream for the hospital, this should be the
response:
• The hospital treats patients and mainly collects payments from the
insurance companies based on the service provided.
• Key facts to be provided to the interviewee if asked:
• Volume of patient change: No
• Any change in mix of services performed by the hospital (lower of number
of surgeries etc.): No
• Key Expected Insight: In some way the revenue per patient on average is
decreasing, which implies that the patient mix is changing in terms of
revenue . Which implies that the patient mix in terms of insurance they cover is
changing
• If patient asks to explain the insurance coverage, provide the following facts:
Primarily the patients can be categorized into the following categories:
• Private Insurance Covered Patients
• Public Insurance Covered Patients
• No Insurance Covered Patients (Usually homeless people)
E-COMMERCE IN AIRLINE INDUSTRY

• The interviewee should now say “so what” and talk about one or
both of:
• There will be $10MM annual return for $100MM investment. Without

discounting, we know it is a 10 years till breakeven. In the software


industry that’s unacceptable
• $10MM till perpetuity. Discount rate for the software industry is high.

Let’s assume 20%. $10MM/20% = $50MM.


• So what? Meaning that the moment you investment the $100MM it

is worth $50MM. this is a bad investment.


• A interviewee can be confused and try and use the airline

discount rate (which is extremely low). This is obviously a mistake as


discount rate is directly correlated to risk.
E-COMMERCE IN AIRLINE INDUSTRY

2. Calculating the cash flow for customer


• $-20MM investment (licensing fee).

• Revenue stream (cost savings)

• $50 transactions will only cost $15 (=$9+$6).

• Savings of $35 per transaction

• 500,000 transactions.

• $35*500,000 = $17.5MM savings a year.

• So what?

• The investment in the software for the customer could be


extremely good investment as it is returned in less than a year and
a half.
• However, first customer can be something else all together as
there are risks than can decrease the returns:
• Not all suppliers will join right away or at all.

• The software will have bugs in the first few months /years.

• The software company can go bankrupt, taking our licensing


fee away and never give any returns.
E-COMMERCE IN AIRLINE INDUSTRY

Recommendation
• The recommendation should include the following:
• The answer –   Candidate suggests that investment in the company
is not a good option as it takes 10 years to recoup investment
(without discount rate). Being a customer looks good, though not
necessarily first customer.
• The number(s) –   NA
• Risks or considerations – 

• Joining as a first customer will be considered if the client will be

compensated for the risks.


• After few airlines will join the system, and the software proven

itself, the client should definitely join as it will save costs.


• Next steps –  NA
E-COMMERCE IN AIRLINE INDUSTRY

Questions to Further Challenge the Interviewee


The following question tests creativity further:
• Let’s assume there is another feature in the software that allows airlines to view
other airlines‟ inventory by location and part. So, for example, if I’m southwest and I
have a plane down in NYC. I contact the supplier and get an answer that I’ll have
the part in three days. I can look into American Airlines inventory and see if they
have the part in NYC. If they do, I can buy it from them at cost and ask the supplier
to send the part their way. Is it a good feature?
• A full answer (push interviewee till he reaches all conclusions or gives up):
• The feature is great for the industry, as the entire industry will be able to reduce
downtime and share the value in its companies‟ bottom line and with the
customers.
• However, the risk is that the feature actually encourages the airlines to carry no
inventory (why should I carry inventory when everyone shares the benefits?)
• A solution to mitigate the risks through the system is to create a system of
checks and balances and create a part exchange account for every airline.
So every time an airline takes a part from another airline then the airline's part
exchange account is reduced by the part cost. Every time an airline provides a
part to another, its parts exchange account is increased by the part cost. If we
cap the part exchange account to say minimum of -$5MM, then we solved the
problem.
• This will actually encourage better inventory management throughout the
industry as airlines will not want to stay away of this program as it provide huge
cost benefits.
CASE 10:
NEW RUBBER PLANT INVESTMENT
Firm Style Interview Round
BCG, Bain, A.T. Kearney, Deloitte 1

Case Question:
The federal government of a country in certain part of the world is investigating whether to
restart a rubber factory in the western part of the country. The factory was operational in
past but has not been used for 7 years. The plant was closed due to terrorism in the area
which has now come down significantly though there are still issues and skirmishes reported
in the area. If rejuvenated the plant may become a target for the rebels. All the equipment
is considered useable but the government still estimates to spend $12M to rejuvenate the
plant which would enable the plant to produce up to 10M lbs of rubber per month. The
demand of rubber worldwide is strong but rubber must be transported to an export port via
trains; up to 2 trains per day can used for this.
Clarifying Questions & Answers
Provide the following answers only if the interviewee asks the corresponding questions.
Question Answer

What are the raw materials? Need gum resin. 3lb of resin after processing
results in 1lb of rubber
Where are the resins coming from? They need to be transported from the capital.
Up to 4 trains can be used for the same (This is
a key question, a candidate not asking the
NEW RUBBER PLANT INVESTMENT

Recommendation
The recommendation should include the following:
• The answer  –  Go ahead with investment in the plan as it seems
highly profitable
• The number(s)  –  With production at 5M lbs of rubber we make a
profit of $12M a year. Production is limited by transportation, an
area that can be looked at and addressed. This should further
increase our profits in future.
• Risks or considerations  –  We have highlighted a lot of the risks, key
is that the government takes steps to mitigate the risks , the
government can take some steps based on our analysis of the
mitigation. Some of the steps could mean a long term
investment.
• Next steps  –  Assess how plant can be staffed, whether
transportation bottleneck can be alleviated, the level of terrorism
threat and steps to mitigate.
CASE 13:
CAPITAL INVESTMENT FOR UTILITY
Firm Style Interview Round
Booz, BCG 1

Case Question:
A major East Coast, vertically integrated and regulated electric utility has
received a permit to build its first nuclear power plant. It wants to know if this
will be a good investment and possible risks associated with this venture.

Clarifying Questions & Answers


Provide the following answers only if the interviewee asks the corresponding questions.

Question Answer

Can you describe the structure of the A vertically integrated utility owns the entire
company? What does “vertically integrated” value chain –  generation, transmission and
and “regulated” mean? distribution. It is somewhat of a monopoly and
is overseen by state regulators.
What is the competition like? There is virtually no competition because it is a
regulated industry.
Cont’d on next slide
CAPITAL INVESTMENT FOR UTILITY

Clarifying Questions & Answers


Provide the following answers only if the interviewee asks the corresponding questions.

Question Answer

What is the size of the plant? The new plant is expected to supply about
8,760 GWh (Gigawatt hours) per year and it is expected
to be utilized for 30 years.
What is the current demand? The current annual demand in the utility’s service area is
17,000 GWh. For this case assume this demand will
remain constant in the near future.
What is the expected revenue on a $80/MWh (1000 MWh = 1 GWh)
variable basis?
What is the expected variable cost? $10/MWh

Is there any public opposition? No

Why is the utility building this new Great question. Currently the company has about 9,000
plant? GWh of supply that comes from old, dirty and inefficient
coal fired power plants. It plans to retire them in the
near future.
Cont’d on next slide
Case 12:
12: Mining in Brazil

BACKGROUND

Firm: BCG
Round: 2007 Summer, Second
Content: Qualitative and quantitative

CASE QUESTION

Our client is a US industrial conglomerate, with major investments in South America,


India and China. One of these investments is a mining operation in Brazil. At this mining
operation, our client produces only one metal, which is considered to be an international
commodity product. This metal has hundreds of applications. In Brazil there are only two
other producers.

The CEO has hired BCG to help identify new opportunities for this business as well as
understand the market dynamics. He wants to know whether he should divest the mining
business or invest in an additional facility. This afternoon, the BCG team is going to meet
with the CEO to discuss
discuss our initial hypotheses.
hypotheses. We have been provided the following
information to assist us:
The interviewer was expecting the interviewee to first provide a structure/framework,
which would look at the big picture and then start hypothesizing. A strong candidate
would:
- Discuss market dynamics (local and international and supply/demand).
- Discuss the expected competitive response to any action, e.g., a price war.
- Summarize all findings in a presentation
p resentation format.

Key facts: All given beforehand, but the following would help in the discussion:
o An efficient plant should have a 1,000,000 ton capacity (but, not all plants are
operating efficiently), however, from this information the interviewee should be
able to assume that therefore competitors are operating more than one plant each.
o Market grows with GDP.
o There is a strong demand for this product internationally.
o The competitors are probably located away from the coast, adding transportation
costs.

EXAMPLE DIALOGUE

Interviewee: [Summarize the case and work on a framework] . In this case it is important
to look at the competition (specifically, understand the different cost structure of the 3
producers), estimate the market demand and discuss the international trade environment.
Interviewee: This is a good point. You did mention that there is a strong demand
worldwide. Now we have to find out why the competitors are not selling their full
capacity. We can think of many possible reasons. Geographical distance, transaction
costs, transportation costs, export taxes, etc.

 Interviewer: They all make sense, but as we see competitors are able to export some
metal, right?

Interviewee: Competitors might have operations abroad so it makes easier to export to


their international facilities. Or they might produce part of their capacity close to harbors.
Considering that the international price is much lower than the local one, I would expect
some barriers for international trade.

 Interviewer: That is correct, Brazil does have some taxes for foreign products and
 producers struggle with transaction costs. Let’s look at the cost structure. Why do you
think there is a difference in costs?

Interviewee: I could consider geographic location, technology, economies of scale,


supply chain synergies, etc.

 Interviewer: You mentioned some important elements. Our client has only one location
and the competitors have multiple plants, each in the countryside. Our client was the first
 player in this market,
mar ket, having built its facilities 10 years
yea rs earlier
ea rlier than its competitors. The
industry has realized that a plant must have a minimum of 1,000,000 capacity in order to
Interviewee: We should be cautious because a lower price would impact the current
profitability. The current margin is ($600 – $450) * 600,000 = $90M. For instance, if we
drop the price to $513/ton we would be losing $52.5M [($600-$513) * 600,000] from the
current 600,000 ton production but generating $93M [($513 - $420) * 1,000,000] from
the new production, a net gain of $40.5M. With the 10% discount the minimum profit
required from the new plant would be $40M ($400M*10%). Therefore, the minimum
price would be $513.

 But if you drop the price to $513/ton,


 Interviewer: $513/to n, what would
w ould the competitors’ reaction
rea ction
be? Would they also increase production? Remember, they have idle capacity and you
don’t.

Interviewee: That is correct, but the competitor has a lot more to lose with a price
reduction. In our client’s case we found out that it would lose money as the margin of the
current production drops. However, the client only sells 600,000 tons right now while the
competitors sell 4,800,000 tons combined. They would probably reduce their production
to avoid a higher price reduction.

 Interviewer:  Really? So you are recommending our client to invest $800M in a


2,000,000 tons capacity plant?

Interviewee: I haven’t done the math but I guess this would be too risky. I would
recommend our client to invest the $400M and see how the market reacts.
Case 25: Stew’s Connections
Connections

Market Entry BCG Round 2  


Our client is a start-up with the ability to deliver broadband internet to commercial airlines. How
Prompt would you help them think about their offering?

 About the case: This


This is a market entry case
case where candidate are required to evaluate the feasibility
of a new product in conjunction with the airline industry. The candidate should use a comprehensive
framework, walk the interviewer through it and be prepared for analytical detours throughout the flow
of the case.

The calculations represented here are only one approach and interviewees may take other 
approaches, depending on the assumptions made. The interviewer should be mindful of th is and 
allow for flexibility.
flexibility.

Industry & Market Size


Size Discussion: The interviewee should have mentioned this as a major bucket
in his/her framework.

Use the information below to provide guidance as necessary.


necessary.
Guidance
Broadband for the airlines
There is general interest in broadband internet from the airline industry. The start up would have to
invest relatively little up front and would keep most of the revenues. They would charge the
customers on a per flight pricing model.

Size of the Market 


 Ask the candidate to estimate the market
market size and hand over Exhibit 1. Inform the candidate that
there are 3,000 planes.
planes. Full answer in chart below 

Pricing 
In order to finish the market size, the candidate should ask for the price per 
flight. Hand out Exhibit
Exhibit 2 and ask candidate to set the price.
Case 25: Stew’s Connections
Connections

Market Entry BCG Round 2  


Class First Coach
Seats/Plane 20 180
Load Factor .75 .75
Full seats/plane 15 135
Analysis Biz Travelers 100% 30%
Laptop users/plane 15 40.5
Total laptop us
users/plane 55.5
3000 planes x 2000 legs/plane x ~50 laptop users/plane = 300,000,000 approximate annual
 potential user-legs.
user-legs.
Case 25: Stew’s Connections
Connections

Market Entry BCG Round 2  


 Assume 100 passengers
passengers (for ease) at the various price/penetration combinations.

30 users at $5=$150/flight

25 users at $10=$250/flight
$1 0=$250/flight

Analysis 20 users at $15=$300/flight


$1 5=$300/flight

10 users at $20=$200/flight
$2 0=$200/flight

5 users at $25=$125/flight

Set price at $15.


Breakeven Analysis
Given the information already revealed in the case and the information below the candidate should
calculate a break-even point.
Guidance
Information to be given if asked:
The company has discovered that if they can generate $250,000 per plane in annual 
revenue, they will be profitable installing the technology on that plane.
250,000/2,000 legs/plane = $125/leg

$125/$15 about 8 users/leg


Analysis
50 laptop users/leg, and at $15, there‘s a penetration rate of 20%, so we estimate 10 users/leg.

Response: Yes,
Yes, they should break even.
Case 25: Stew’s Connections
Connections

Market Entry BCG Round 2  


Other factors
Guidance Probe the candidate for breadth and understanding of new market entry. Ask him/her about the
following aspects of this project .

Competition
The interviewer should probe deeper into the competition especially with
with regard to Intellectual
Property.
Property. For this case, the company has the patent on the high speed connection.

Analysis What about low-speed internet connections?

Risks
 Ask the candidate which risks are associated with
with the business model. Use your 
 judgment when
when considering their answers.
Candidate should provide a crisp recommendation –
recommendation – A  A crisp recommendation
recommendation should be roughly 30 -
45 seconds long and should include clear bullet points
points that support an overall recommendation.
recommendation. For
example, ―Enter the market for reasons 1, 2 and 3‖

Expected: Accurate arithmetic with


with solid profitability and breakeven calculations.
calculations.
Performance
Good: Candidate provides a framework that includes exploration of all possible options and offers a
Evaluation
clear recommendation that summarizes key findings in under 30 seconds.

Excellent: Candidate considered sufficient number of other factors while maintaining poise through
―what else‖ line of questioning at the end of the case.
Case 25: Stew’s Connections
Connections

Market Entry BCG Round 2  

Exhibit 1
Case 25: Stew’s Connections
Connections

Market Entry BCG Round 2  

Exhibit 2
Case 15: Convenience Store
Profitability BCG Round 1
Our client is a major convenience store chain with 5,000 stores in the US and $25B in annual
revenues. About one year ago their CEO hired our firm to help increase profits. Since then, we
have worked with the client to aggressively control costs through negotiating larger discounts from
Prompt suppliers, restructuring the client‘s labor force, and cutting other operating expenses by 10%. As a
result of our work, our partners believe there is little room for cost reductions going forward. How
else can we increase profits for our client?

1) B ackground data (TO BE GIVE N ONLY IF ASK ED) :


• There are 4 major companies (including the client) in the industry that control 95% of the market.
Each has a roughly similar share of the market.
• Client is not interested in expanding/changing product line.
• Client does not want to expand internationally due to logistical concerns.
• We studied possible M&A activities, but there are no desirable targets.
• Client charges same price per item in each of its stores. (i.e. no difference between prices of one
item in Manhattan versus Durham)
• Consumers consider Motrin and Tylenol to be very different brands, with each having very loyal
customers. It would take a substantial change in price of one brand to convince customers to
switch to the other. Buyers of both brands would consider the store brand product , however
Guidance they feel that it is a slightly inferior product and would need to be incentivized to do so.
•  Advil and Motrin suppliers charge the same price to every convenience store chain
•  All other costs (overhead, etc.) should be considered the same across the 4 major chains
 2) C ase delivery : Initially try to get candidate to brainstorm about how they might be able to
increase profits. Present candidate with Exhibit 1 if they inquire about pricing, products sold, or
sales volumes. Depending on whether candidate wants to focus on Food&Bev or Pain Relievers
first, present them with Exhibits 2 or 3 respectively.
Case 15: Convenience Store
Profitability BCG Round 1

Exhibit 1: Candidate should immediately focus on Pain Relievers and Food &Beverage segments.
Pain Relievers have lowest margin and are most inelastic, which suggests opportunity to increase
prices. Food&Bev is our best seller in terms of Sales and Scan Margin, so look for ways to sell
more or gain share. Cleaning Product sales are $4B. Price elasticities are meant to be illustrative
and are not needed for any calculations. (Note: Scan Margin is essentially the same thing as profit
margin, however it also includes funding from producers. It is a term frequently used by retailers).

Exhibit 2: Candidate should recognize that client has highest unit price and lowest volume/share
relative to competitors in Food&Bev segment. This would imply that there is little room to increase
prices in this category. He/she should recognize that this is not the solution and should go explore
Pain Relievers .

Analysis Exhibit 3: Candidate is presented with Total Dollar Sales, Price and Cost data for each of the three
brands of pain relievers. He or she can then calculate profit per unit, total unit sales, and total
$profit per brand

Exhibit 4: The key insight is that our client charges a lot less for Advil that the competition. We
charge the same for Motrin and slightly more for Store Brand. A good candidate will inquire about
market share data at this point, and should be presented with Exhibit 5

Exhibit 5: Candidate should determine that we have a very strong share in Advil (which accounts
for over 50% of industry-wide pain reliever sales). This combined, with our significantly lower price
and profitability in Advil, as well as slim likelihood that customers would switch brands or to our
competitors, means that we should raise prices.
Case 15: Convenience Store
Profitability BCG Round 1
Expected:
•Candidate recognizes that we need to focus on revenues going forward
•Identify that revenues are a function of price, quantity, and mix and ask for more data
on each component
•Recognize that priority should be on investigating Food&Bev and Pain Relievers
when viewing Exhibit 1 and ask for more information on each
•Determine that raising price of Advil is the solution to the case

Good:
• Quickly determine that Food&Bev and Pain Relievers drive profitability
• Candidate performs calculations quickly and accurately
• Recognize that Food&Bev is not the solution after viewing Exhibit 2
Performance • Candidate will ask about competitor prices and market share data when viewing
Evaluation Exhibit 3

Excellent:
• Quickly recognize that client has highest unit price and lowest volume/share relative
to competitors in Food&Bev segment. No need to perform market share calculations
• Inquire about shifting share to Store Brand to generate more profits (since it has
highest profit per unit)

Deeper Insights (Second Years)


• Attempt to determine profit impact of increasing price of Advil
• Have insight that charging higher prices for same item in certain markets would be
Case 15 : Convenience Store
Exhibit 1 - Products Sold by Client

Profitability BCG Round 2

Total Client Sales Total Scan Margin Price Elasticity

Pain Relievers $10 B 0.3

Cleaning Products $0.5 B 1.1

Food & Beverage $9 B $3 B 1.4

Other $2 B $0.4 B 1.8

Total $25 B $4.9 B


Case 15: Convenience Store
Exhibit 2- Convenience Store Industry Food &
Beverage Sales
Profitability BCG Round 2

Average
Total Unit Volume Total $ Sales Share
Price/Unit

Client $1.50 $9 B

Competitor 1 10 B $14 B

Competitor 2 $1.35 12 B

Competitor 3
$1.45 8B

Total 36 B $50.8 B
Case 15 : Convenience Store
Exhibit 2 ANSWER KEY  – DO NOT SHOW
CANDIDATE
Profitability BCG Round 2

Average Total $Sales Share of Total


Total Unit Volume
Price/Unit Volume Market

~18%
Client $1.50 6B $9 B

$1.40 ~28%
Competitor 1 10 B $14 B

$16.2 B ~32%
Competitor 2 $1.35 12 B

Competitor 3 $11.6 B ~22%


$1.45 8B

Total 36 B $50.8 B
Case 15: Convenience Store
Exhibit 3 - Client Pain Reliever Sales

Profitability BCG Round 2

Total $
Price/Unit Cost/Unit
Sales
 Advil $8 B $3.99 $3.79

Motrin $1 B $4.99 $4.49

Store $1 B $1.99 $1.29


Brand
Case 15: Convenience Store
Exhibit 3 ANSWER KEY  – DO NOT SHOW
CANDIDATE

Profitability BCG Round 2

Total $ Total Total


Brand Price/Unit Cost/Unit Profit/Unit
Sales Units Profit

 Advil $8 B $3.99 $3.79 $0.20 ~2 B $0.4 B

Motrin $1 B $4.99 $4.49 $0.50 ~200 M $0.1 B

Store
$1 B $1.99 $1.29 $0.70 ~500 M $0.35 B
Brand
Case 15: Convenience Store
Exhibit 4- Convenience Store Pain Reliever Prices

Profitability BCG Round 2

Brand Client Price Average Competitor Price

 Advil $3.99 $4.69

Motrin $4.99 $4.99

Store Brand $1.99 $1.89


Case 15: Convenience Store
Exhibit 5 - Pain Reliever Market Share $ by Brand
Profitability BCG Round 2

18 10 7

10%
14%

Client 44%

90%
86%

3 Competitors 56%

 Advil Motrin Store Brand


Case 28: Stew’s Connections

Market Entry BCG Round 2


Our client is a start-up with the ability to deliver broadband internet to commercial airlines. How
Prompt would you help them think about their offering?

 About the case: This is a market entry case where candidate are required to evaluate the feasibility
of a new product in conjunction with the airline industry. The candidate should use a comprehensive
framework, walk the interviewer through it and be prepared for analytical detours throughout the flow
of the case.

The calculations represented here are only one approach and interviewees may take other
approaches, depending on the assumptions made. The interviewer should be mindful of this and
allow for flexibility.

Industry & Market Size Discussion: The interviewee should have mentioned this as a major bucket
in his/her framework.

Use the information below to provide guidance as necessary.


Guidance
Broadband for the airlines
There is general interest in broadband internet from the airline industry. The start up would have to
invest relatively little up front and would keep most of the revenues. They would charge the
customers on a per flight pricing model.

Size of the Market


 Ask the candidate to estimate the market size and hand over Exhibit 1. Inform the candidate that
there are 3,000 planes. Full answer in chart below

Pricing
In order to finish the market size, the candidate should ask for the price per
flight. Hand out Exhibit 2 and ask candidate to set the price.
BIG  R ED C 
 ASE  BOOK  JOHNSON  SCHOOL CONSULTING CLUB

INSURE ME!

Type of case: Strategy 


Company: BCG

Description: Insure me is a Global Financial Services company at the insurance business.


Recently, the CEO of the company was fired and took with him all of the 10 employees of the
company’s private funding division, which was his pet project. No one that is left in the company
knows what is going on in that division, and there is no reporting system to rely on (the CEO took
all of the data with him). How would you go about managing this division?
Additional data:
 The company is operating in the US and Europe.
 The company provides car, life and other type of insurance.
 The company is one of the 4 leading players at its market with over $1B of annual
revenues.
 The private funding division is type of a VC.
 We have a data sheet (see appendix) which list 4 of the division’s current investment.
 These 4 investments are only around 20% of the number of investments but form 80% of
their value.
Solution Structure:
 Identify the company’s business and core competency.
 Identify the assets under the division management.
 Identify any financial and strategic synergies between the division’s assets and the
company.
  Analyze ways to leverage the division and its assets moving forward.
Solution Analysis:
  As mentioned the company’s core competency is in the insurance field.
  As could be observed from the appendix two assets are not complimentary to the
company’s business.
BIG  R ED C 
 ASE  BOOK  JOHNSON  SCHOOL CONSULTING CLUB

Name of A B C D
company
Field High Value Stadiums Golf clubs Executive
commodities renovation design insurance
insurance
This year’s $150M $300M $100M $70M
revenue
This year’s $100M $280M $150M $50M
expenses
Next year’s 300% 200% 100% 300%
revenue growth
(additional on top
of the current)
Next year’s 500% 200% 150% 400%
expenses growth
(additional on top
of the current)
Too Many Books: Storage and d istribu tion dilemma

Context

Books Galore is a company in the publishing industry with stable sales, in regards to both
volume and value.

Its distribution warehouse, which is used for storage and order preparation, is reaching
maximum capacity, resulting in a rapid deterioration in the quality of service.

The Head of Books Galore’s distribution subsidiary wishes to build an extension to the
distribution warehouse, at a £10m cost, though the project would bring no additional
revenues.

The CEO has asked you to determine whether the investment is necessary or whether it can
be better used elsewhere

Question 1
These three possible explanations indicate causes for the increasing space usage and
possible levers to address them:

Number of Overall warehouse


Number of volumes /
capacity
SKU
SKUs / Titles

 Overestimate of

  Many new titles potential sales


Potential Causes for   New legislation
increased space
  Old titles not   Longer

usage   Safety issues


destroyed replenishment
cycle

 Eliminate part of

overstock

 Reduce initial

stock   Extend the


  Eliminate number
warehouse
Potential levers to of titles stored
 Reprint on request
 (client’s
increase capacity
proposed
  Shorten

solution)
replenishment
times
• Space released
• Economic impact

Question 3

How could you assess the impact on the warehouse P&L statement of a
particular publisher’s exit?

The impact on profit would be the changes in revenue minus the changes in costs.

Let’s first proceed with costs – how would you analyse the impact on costs?

Total costs are composed of:


• Fixed costs (independent of the number of publishers)
• Variable costs (decreasing when a publisher leaves)

In this example, fixed costs would be the building itself, sets of shelves and other
infrastructure, as well as some of the administrative and IT staff that coordinate operations in
the warehouse.

Variable costs would include handling staff and packaging.

I would like to determine how much each of these costs contributes to the overall expense of
running the factory.

 After investigation, you establish the following cost structure:

10
Question 4

What are the drivers of our v ariable costs?

Variable costs are encompassed by handling staff and packaging. In terms of time utilisation,
that requires filling title / SKU spots, fetching one or several volumes from these places, and
then packing these books for distribution.

Cost drivers for these actions would be:

• Number of movements to be performed = number of times operators have to


fetch one or several books of a same type, which means that personnel
expenses will vary depending on the number of order "lines"
• Packing - Number of boxes / packages, which corresponds to the number of
books (assuming that the size and type of book does not change)

This gives us a good idea of potential areas for cost savings. But what about the impact of
decreased number of publishers on revenue?

Question 5

If  revenues are based on the price of the book (fixed at 5%), which
publishers sho uld be terminated?

Our goal should be to maximise profits through a publisher’s exit from the warehouse. Since
revenues are a function of the price, two possibilities would be:
• Eliminating publishers with low average price and rotation
Expected revenue would be £18m

Total costs currently £15 (= sales of £20m – margin of £5m)


Current cost breakdown:
• Fixed (50%) = £7.5m
  Personnel (40%) = £6.0m

• Packaging (10%) = £1.5m

Projected cost without publisher:


• Fixed = £7.5m (no change because fixed cost)
• Personnel = £6.0m – 20% = £4.8m
• Packaging = £1.5m – 10% = £1.35m

Total costs wit hout publisher expected to be £13.65m

New margin = expected revenue – expected cost


= £18m - £13.65m
= £4.35m

Margin would thus decrease by £0.65m by eliminating services for that publisher

Question 7

If eliminating that publisher would reduce margins by £0.65m per year,


what is our best option - terminating a publisher’s contract or building
an extension?

To assess which is best, we could compare the potential annual loss of eliminating this
publisher with the cost of the investment proposed initially
Whis key Brand Turnaround
Context

The client has been in business close to 90 years. The original founder started in the whisky
business, although, over time, successive leaders expanded the product line so that today,
the company is a multi-billion dollar diversified liquor manufacturer, with less than 5% of its
sales in .

This expansion strategy made particular sense over the last 20 years, as total U.S. sales
have been declining by ~2% per year, and the client’s brand has been declining at the same
rate.

While the company has enormous passion for the brand (as the founding basis of the
company), in recent years they have been paying limited attention to the brand. Last year,
however, events caused our client to suddenly pay attention to their brand. While the
market declined by 2% again, our client’s brand declined by 15%, despite selling 10m bottles.

The client comes to you because they A) want to find a way to grow the brand again; but B)
want to do this without lowering the total profits they were taking out from the brand.
To establish which of these factors is decisive in the decline of the brand, further examination
will no doubt be required.

Some members of the board have voiced their concern about our marketing spend vs.
competitors, but company marketing spend hasn’t changed in the past several years.
Perhaps competitor marketing has changed significantly.

Question 2

How could we determine if there has been change in relative marketing


spend against major competitor s?

First we’ll need to gather information on our own marketing spend, so that we have a basis of
comparison.

Secondly, we need to determine our competitors’ marketing spend, which we could do


through several sources:
 Internal interviews
 External benchmarking through competitor company reports
 Count of number of ad pages in magazines
 Interviews of experts within the advertising and marketing industry

Interviews with advertising experts and an examination of competitors’ ad pages have shown
that while marketing spend hasn’t changed significantly, there has been significant change in
prices within the industry. The following profitability data was gained from the above
investigations:

Place of
Cost to
Brand 2005 price 2006 price 2007 price
Question 3

What conclusions can we draw from this client and competitor


profit ability data?

It appears as though competitors A and B have been slowly increasing their prices on
Premium brands, aiming at a segment of consumers who are willing to pay more for their at
specialty stores. It is possible that they have succeeded in capturing a more lucrative share
of the market

 At the same time, they have introduced ‘Own Brand’ labels for selling in grocery stores that
are significantly cheaper than the client brand.

Thus, it seems that competitors have been capturing the lucrative top end while also
launching new brands that have captured the low price-end, effectively squeezing the client
brand out of the market.

Considering our production costs and the price of Competitor A and Competitor B ‘Own
Brands’, we cannot compete on price in supermarkets. With our strong brand, we must aim
to recapture the high end of the market, hence increasing prices.

Question 4

What price should w e set and how will it imp act profits?

In order to understand the impact of an increase in price on profits, I would need information
on the number of bottles sold and some info on the cost structure.
Total costs would therefore be £68M, yielding a total profit of £92M. A higher price would
therefore position our brand in the higher-end market, decreasing the amount produced but
increasing profits by £22M with a higher price.

Question 5

What complications might arise in our attempt to capture this higher-


end market?

In the short-term, we need to be aware of how competitors may react to the move, by
adjusting their pricing, branding or advertising. Our move could provoke a change in prices
amongst competitors that will leave us positioned as before. We should also be aware of
possible difficulties in marketing our brand in the premium category as the third mover.

Further complications could arise given that our brand exists across a wide portfolio of
products. The price increase of our could negatively impact the customer base for other
products, if the customers who stop purchasing our at a higher price develop negative
feelings regarding the brand.

In the longer-term, we should examine whether it makes sense to re-invest earnings into the
business since it is a declining market. Investments might be most profitably put to use by
growing other parts of our business.
ChairCo  – BCG Round 1

Guidance for interviewer and


Problem statement narrative
information provided upon request
Our Client, ChairCo manufactures metal parts* that  If the candidate asks, tell them that there are no
are used to manufacture chairs. ChairCo primarily specific financial targets.
sells these parts to US based chair manufacturers.  Give the exhibits in the subsequent slides only
They are facing declining revenues and the CEO has when the candidate asks for the relevant data.
asked us to evaluate the problem and suggest
corrective measures.

*Metal bases that are used in the revolving office


chairs.

Difficulty: Easy Industry: Manufacturing Type: Profitability, Operations, Chart Based

- 37 -
ChairCo – BCG Round 1

Additional Questions to Steer Discussion

Questions for the candidate

 After seeing Exhibit 1, the candidate should make an observation that prices and volume are
decreasing and both these issues need to be addressed.

 Why did ChairCo have to decrease prices? Because competition has decreased prices.

 Why did competition decrease prices? Because metal parts are a commodity and they might
have a lower cost structure than us.

 Why do you think our competitor has a lower cost structure? Material and labor could be the two
major reasons.

 Why is our client loosing unit sales despite decreasing price? Because their customers are
moving to low cost countries.

 Can the client reduce costs? Client is already very efficient and cannot decrease their costs
without shifting operations to China, Indonesia etc.

- 38 -
ChairCo – BCG Round 1

Suggested Solution and Structure

Solution Guide

 Exhibit 1 – Volumes have decreased and so have prices ($10 to $9.5). Ask candidate why he/she
thinks the price must have gone down. The most logical answer should be that since this is a
close to commodity product, prices for the entire industry have f allen down and ChairCo had to
respond. Competitors might have become more cost competitive because their operations are
located outside US.

 Exhibit 2 - Competition has significant cost savings in material and labor. The most logical
reasons are that they are based in low wage counties such as China, Indonesia and that they are
using an inferior/cheaper metal.

 Exhibit 3 – ChairCo customers are moving geographically away which explains the drop in
volume despite the drop in price.

- 39 -
ChairCo – BCG Round 1

Conclusion

Recommendation Next Steps

To become cost competitive and gain proximity to   Analyze which country has low cost
customers (chair manufacturers), ChairCo has base, high proximity to customers and
to shift manufacturing to Asia. low barriers (regulations, etc.) to set up
manufacturing.
Risk – downsizing in US will lead to a PR
backlash.

- 40 -
ChairCo – BCG Round 1

Exhibit 1  – ChairCo Sales

$500 M
50M Units
$380 M
40M Units

2010 2011

- 41 -
ChairCo – BCG Round 1

Exhibit 2 – ChairCo Vs. Competitor cost

ChairCo Costs
Cost Structure
SGA COGS
ChairCo Competitor

Materials 4.9 2.5

Labor 2 1.5

Transportation 0.5 1.5


$7.5 $7.4

Tax 0 1

IT 0.5 0.6 $1.5 $1.6


Overhead 1.1 1 2010 2011

- 42 -
ChairCo – BCG Round 1

Exhibit 3 - Manufacturers of Finished Chairs selling in US

Europe Canada US Asia

10.0%
30.0%
50.0%

80.0%
60.0%
40.0%

5.0% 5.0% 5.0%


5.0% 5.0% 5.0%
2009 2010 2011
- 43 -
Midwest Hospital  – BCG Round 2

Guidance for interviewer and


Problem statement narrative
information provided upon request
Midwest Hospital is a research-based hospital and If the candidate asks tell them that there are no
takes pride in its joint replacement surgery financial targets.
department. Recently they did a P&L analysis for all Give the exhibits in the subsequent slides only when
departments and found that the joint replacement the candidate asks for the relevant data.
surgery department is making losses. The CEO has
asked for our help. Candidate should figure out during the course of the
case that there are several levers that can increase
profitability:
1. Increase price
2. Change patient mix
3. Increase total number of surgeries
4. Decrease costs
5. Provide post surgery services such as
physiotherapy (vertical integration)

Difficulty: Hard Industry: Healthcare Type: Profitability

- 70 -
Midwest Hospital – BCG Round 2

Additional Questions to Steer Discussion

Questions for the candidate

 At some point near the start of the case, interviewer should take the lead and ask these questions
after exhibits has been given

1. Exhibit 1: Would it be advisable to not cater to Medicare patients (assume no backlash)?


2. Exhibit 2: What is the number of surgeries that Midwest needs to conduct in a year to
breakeven?
3. Exhibit 3: Why is Company D able to stay profitable despite having fewer patients and
unfavorable patient mix?

- 71 -
Midwest Hospital – BCG Round 2

Suggested Solution and Structure

Solution Guide

1. On fully cost allocated basis Medicare patients are unprofitable but they are still paying $1K
above the variable cost (marginal cost). This helps cover the fixed costs of the department.
So, it is not recommended to stop conducting surgeries for Medicare patients.
2. Average revenue per patient is 19K. Average variable cost is 14K. Gross margin per patient is
5K. Fixed costs are 7M, so 1400 surgeries are required for breakeven. Assuming same
proportion as in Exhibit 1 the hospital requires 140 commercial, 420 insurance, and 840
Medicare patients.
3. Comp D might have a lower cost structure or are able to negotiate better pricing from payers.

- 72 -
Midwest Hospital – BCG Round 2

Conclusion

Recommendation Next Steps

 Increase total number of patients.   Analyze scope for cost reduction, starting with
 Change mix of patients to have a higher competitive benchmarking.
proportion of commercial and insurance   Analyze scope for increase in price, starting
customers. with competitive benchmarking.
  Analyze profitability of post care services
provider.

- 73 -
Midwest Hospital – BCG Round 2

Exhibit 1: Patient Mix

Payer Type # Surgeries List Price Invoiced price

Commercial (Enterprises) 100 $40,000.0 $40,000.0

Insurance 300 $40,000.0 $20,000.0

Medicare (Government ) 600 $40,000.0 $15,000.0

- 74 -
Exhibit 2: Joint replacement department P&L

Revenue 19M

VC Physician 5M
Materials 5M
Others 4M

FC Facilities 3.5M
Others 3.5M

Costs 21M

Profit (2M)

- 75 -
Midwest Hospital – BCG Round 2

Exhibit 3: Competitive Benchmarking

Surgeries Commercial HMO Medicare Profitable

Midwest Hospital 1000 10% 30% 60% No

Comp A 1200 20.0% 20.0% 40.0% Yes

Comp B 800 30.0% 20.0% 50.0% Yes

Comp C 900 10.0% 20.0% 70.0% Yes

Comp D 1000 5.0% 25.0% 75.0% Yes

- 76 -
University of Michigan Business School
Case Book 1 - 2001-2002 Edition
Michigan

Practice Business Case 1


Company: BCG

1. Our client, AEM Inc, is an agricultural equipment manufacturer, with 4


product lines, facing declining sales. We have been brought in to examine the cause
and recommend initiatives to improve their profitability.

  Interview Hot Tips: This is a profitability case

There are two broad drivers contributing to d eclining sales for AEM – decreasing unit
sales, decreasing unit price, or a combination of both of these drivers. To procee d, we
would examine the macro and micro perspectives to understand the reasons for declining
sales. For the micro perspective, we would need to understand details of AEM’s business.
For the macro perspective, we would want to study the changes in the industry over the
last few years due to new competitors, emerging technology, changes in consumer
demands, and other factors.

Firstly, we study AEM and its sales trends across the 4 product lines.

⇒ The sales trends for AEM across its product lines are detailed in this exhibit.

Sales Sales % Gross Margin


Product Lines 1995 1996 1997 1997 1997
Direct Combines $ 289 m $ 267 m $ 235 m 67% 43%
Tractortronics $ 73 m $ 71 m $ 65 m 19% 21%
Crossmaries $ 25 m $ 28 m $ 34 m 10% 17%
Hewittrangers $ 15 m $ 15 m $ 16 m 5% 19%
University of Michigan Business School
Case Book 1 - 2001-2002 Edition
Michigan

The industry has seen an emergence of a new technology in Direct Combines with
 John Drill leading the initiative. This has been the primary driver for change in
consumer demands. We should not worry about Tractortronics since is a declining
marketplace and its products are being replace d by Direct Combines.

The last statement above is a strong hint from the interviewer to ignore Tractortronics in
our further analysis, and focus the discussion on the Direct Combines product line. This
emergence of the new technology explains John Drill’s increased market presence – we
can now see that this new technology in Direct Combines may be a primary driver for the
market share shift. The next step is to study the impact of this technology on the industry
and AEM – is the technology here to stay, how mature is this technology, how
sustainable is the advantage due to this technology and what are its switching costs for
AEM. This will help us establish if the technology is worth adapting for AEM. If it is
worth adapting, we need to address how would we implement this technology change for
AEM’s products – in terms of switching costs, new product development, marketing and
 promotion initiatives, and other issues. If the technology is not worth adapting or cannot
 be adopted (due to patent reasons, for example), then we can examine discontinuing the
Direct Combines product line over time.

This technology helps significantly reduce maintenance costs and maintenance


related downtime of agricultural equipment by enhancements to the engines and drive-
train. Given our interviews with AEM’s engineers and marketing managers, as well as
industry analysts, we understand that this technology is becoming a market standard and
will be here to stay.

We recommend that AEM introduce the new technology in its Direct Combines range of
 products. Given that this product line is important of and
University of Michigan Business School
Case Book 1 - 2001-2002 Edition
Michigan

Practice Business Case 9


Company: BCG

9. A regional banking client wants to go into the online brokerage business. What
would you advise them to do?

  Interview Hot Tips:


o This is an open ended, market-entry case. One option is to use the 4C
 framework (customers, competitors, cost, capabilities)
o Stay focused on the question. Interviewer gave a lot of extraneous data on
 several sheets.
i. First and foremost: Should the client enter the industry?
ii. Second: How should the client enter?
o  In terms of prioritization: Understand the market first!

i. How does an online brokerage make money?


ii. What are the relevant segments?
iii. Are there any unmet needs?
o Then look at the competitors.
i. Do they cater to certain customer segments?
ii. What are cost structures?
iii. What competitive advantages do they possess?
o Wrote the 4Cs on a sheet of paper but only to keep track of own thoughts.

Customers
University of Michigan Business School
Case Book 1 - 2001-2002 Edition
Michigan

Well, since we are providing trading services, I don’t think there would be too much
differentiation. Perhaps, some competitors might provide more research capacity.

 Any other ways

Advertising for the most part. We can observe from the info that there has been a huge
increase in advertising. Moreover there are no real differences in product features. How
about differences in cost structures?

What does it mean to you that there is only limited differentiation?

Well, this would imply that demand in this market could be very sensitive to price, which
could lead to a low-margin, high-volume kind of environment. In that scenario, the cost
structure would be very important.

 Interviewer provided all this cost data

Well, we don’t have too much information on that. I don’t think we would have a cost
advantage though. The biggest portion of total operating costs tends to be advertising
advertising.

Capabilities

How do customers perceive the client?

 How do you find that out?


University of Michigan Business School
Case Book 1 - 2001-2002 Edition
Michigan

Practice Business Case 11


Company: BCG

11. Client is a yearbook publishing company that is experiencing declining


profitability. What should they do?

  Interview Hot Tips:


o This is a standard “Profit = Revenues – Costs” case
o  Prioritize issues and filter out unneeded
unneeded information
information - Started to
to query the
revenue side first. Nothing there. Unit prices constant, revenue flat. Next,
explored costs.
o  Framework
 Framework used – “Profit
“Profit = Revenue
Revenue – Cost”
Cost” framework
framework
o Visual Representation - Drew out the manufacturing process and went
through each stage to explore the costs.
o Solution hints provided by interviewer
• Cause: Sales force incentives led to a change in order mix.
• Should change manufacturing process to do small batches.

I would like to explore to the revenue and costs.


 Revenue - Could you tell me what happened to the prices?

 Prices have
have stayed constant
constant in real
real terms.

How about quantity sold?

 Pretty much
much flat.
University of Michigan Business School
Case Book 1 - 2001-2002 Edition
Michigan

At this point I felt I was at a dead-end. I needed some more informat ion about the
client to figure out the problem. Hmmm. Could you explain how the production
 process works so I can understand the cost structure better?

Set-up costs are very high. It is optimal to do longer runs so to lower costs.

Is it therefore some kind of mix issue. Could you tell something about the changes in
demand?

Yes. We have small schools and large schools that use us. The smaller schools are
ordering fewer yearbooks while larger schools are ordering larger quantities. We
therefore started to do more uneconomic batches.

Why are small schools ordering less?

Well it is certainly not the case that the # of students has changed dramatically at
 smaller institutions
institutions.. Can you think
think of some solutions toto increase the order quantity
quantity from
 smaller schools?
schools?

What drives sales?

One key driver: having your picture in the yearbook.

Do the schools order all of their yearbooks at one time?

Good questions. Actually they tend to order them at one, but often call us up after
and ask if they can order just a few more. We have always provided them as a
Eye Glasses (1 of 4)
BCG,
BCG, Roun
Round
d1

Guidance for interviewer and


Problem statement narrative
information provided upon request(1)

The client is an eyeglass (lens) manufacturer from Japan. -They currently have operations and sales in Asia,
They are a well known brand in Japan and are Europe and Canada.
contemplating entering the U.S. market.
- They make the thinnest lens that is also anti-reflective.
This is patented.

U.S. market is a good idea. - Their criteria for entering markets is if they can capture
2% of the market within the first five years and be
profitable.
Eye Glasses (2 of 4)
BCG,
BCG, Roun
Round
d1

Eye Glasses Market Distribution Channels

Somewhere in the student’s framework, they should ask The student should brainstorm channels and you should
about what the U.S. market is like or try to size the U.S. guide them to the following data:
market. You can provide the following information to
help them do this: - Eye care/doctor
care/doctor distribution
distribution = 50% of market
- Mass merchandisers
merchandisers (like
(like Wal-Mart)
Wal-Mart) = 20% of the market
-The U.S. market has one player owning 40% of the - Retail (like lens
lens crafters)
crafters) = 20% of
of the market
, , -
fragmented amongst 10 players
Ask the student what they think of this and if they have
Ask the student what next? any thoughts on which distribution channels the client
They should recognize that 20% of the market is very could focus on?
fragmented and that this = opportunity.
opportunity. They should
focus on penetrating this 20% and should see that Good answers will recognize that the client has a special
logically getting 2% in 5 years is feasible. lens and you’d want to go to the channel in which people
are buying that lens and that mass merchandisers may
A good student will try and inquire about how people have long standing relationships or contracts that make
buy glasses in the U.S. and you will share with them that penetration for a new company difficult.
that (unlike Japan) they are not loyal to a brand, but
rather the place where they go to buy their contacts or In general, guide the student to focus on Eye care
glasses. because it’s here that people buy special lenses most
often and the client would also like to focus where most
A good student will then ask about distribution channels. of the volume is. Dismiss other saying there isn’t enough
info available on this channel.
Eye Glasses (3 of 4)
BCG,
BCG, Roun
Round
d1

Volume Profitability

Once the student focuses on the two channels, they Information on costs and revenues:
should ask what the volume going through Eye care is.
-Tell them its 80MM pairs (this means 160MM lenses). - They sell single
single vision
vision for $25/pair 
$25/pair 
- They sell progressiv
progressivee for $100/pair 
$100/pair 
They should next consider how much of this can the
client capture. They should ask about the different Ask student to brainstorm costs. The cost information

- 50% of total
total volume is single vision
vision lenses -Variable:
- 25% of total
total volume is a “buy and
and try” lenses -single vision: $12.50/pair 
- 25% of total
total volume is progressive
progressive lenses
lenses -progressive vision $50/pair 

The client has products in single vision and progressive -Fixed: can be allocated to single vision at 25% of
lenses. They don’t make “buy and try” lens types (so variable and to progressive as 30% of variable
student should see that the total market size is now 75%
of 80MM pairs or 75% of 160MM lens). -Ignore all other fixed costs right now

The student should now check for profitability


Eye Glasses (4 of 4)
BCG,
BCG, Roun
Round
d1
Now ask HOW they would go about penetrating
the U.S. market broadly? What would
w ould they look Summary
at to actually execute upon this?
Student should see that the company goals on Student recommendation should include:
profitability and market share can be met.
1) Answer:
nswer: Yes, you
you can
can enter
enter U.S.
U.S. market
market and reach
reach
A good answer will look @ the following dimensions: your goals

Focus on Distribution: - We looked at market, and 20% is fragmented, you


-
(mass merchandisers may be locked into contracts and care (which is majority of market)
in eye care/retail you will get more specialty consumers - Looked at costs/revenues and profitability is there
who will be interested in the client’s special product)
- pursue benefits
benefits they could provide
provide to the channel,
channel, 2) Next Steps: Build Distribution, Hire Sales People
maybe they can deliver with better speed or provide
better terms (something that their competitors in the 3) Risks: Brand name doesn’t go a long way and this one
market aren’t doing) of the assets you have. Basis of competition in the
U.S. is different than in Japan and what the client is
Marketing: used to.
- focus on channel,
channel, not on end consumer.
consumer. End consumer
consumer
doesn’t care about brand (unlike client’s home country
and other Asian markets)

Price:
- ensure price is competitive
competitive
New Airline Routes (1 of 4)
BCG, Round 2

Guidance for interviewer and


Problem statement narrative
information provided upon request
This first question requires the candidate to assess all
In the 1960s, PanAm had a monopoly on routes within the potential reasons for wanting to stop in Kansas City.
City.
the United States from the West Coast to the East Coast. A sample of responses could be:
These routes originated from three points on the West
Coast – Seattle, San Francisco,
Francisco, and Los Angeles,
Angeles, with • Using different planes (shorter range)
range) –
 – the same plane
terminating points in NYC, Washington, and Atlanta. is used on all routes.
ee x • –
 passengers are picked up from KC 
The airline was presented with the option to introduce a
• Picking up additional cargo – no additional cargo
stopover point in Kansas City (see Exhibit 2).
• Attempting to open up a new hub – No need; KC is a
small market 
1. Why would the airline want to do this? • Load factors are low; attempting to consolidate new
routes – Yes, load factors are low. Press the candidate
on why load factor is important (used as a utilization
factor for scheduling and purchasing aircraft, also
increases revenue per aircraft)

 All of the above answers are acceptable, but load factor


is the critical area. The candidate should specifically ask
for load factor amount (currently at 50 percent).
New Airline Routes (2 of 4)
BCG, Round 2

Additional questions for candidate Solution guide

Provide candidate with Exhibit 2 and have him/her The candidate should realize that, at 50 percent load
explain the new route network he/she would utilize. factors, planes can be consolidated.

• If the load factor is 50 percent on all of PanAm’


Pa nAm’s
s A shortcut the candidate should use: examine
e xamine all
routes from west to east, what will be their new load three routes from ONE city and extrapolate.
factor from KC to the East Coast if they now stop in
ansas y • ssu
ssum ng sea
sea s per p ane
ane u can e use or
any number of seats), and equal distribution, the new
• What if the load factor was 80 percent? load factor will be 75%.

• At which point does the load factor become •Applying the same approach, the candidate should
irrelevant (i.e., consolidation of a/c will not be discover that the load factor is still 80 percent.
economically beneficial)?
•Once the loads of aircraft exceed the capacity of two
• Based on the above calculations, what does this planes (or 66.7%), then three planes will be needed
suggest about PanAm’s decision? per KC-east coast route.

•What recommendations would you make to  Above a certain load factor, the benefit is moot.
management? Several factors should be considered:
Will load factors increase on west coast-east coast
routes beyond 66.7%? 
 Are additional opportunities for KC expansion
available? 
New Airline Routes (3 of 4, Exhibit 1)
BCG, Round 2

CURRENT ROUTES

SEATTLE

SFO NYC
WASHINGTON, DC

LAX

ATLANTA
New Airline Routes (4 of 4, Exhibit 2)
BCG, Round 2

PROPOSED ROUTES

SEATTLE

SFO NYC
KANSAS CITY WASHINGTON, DC

LAX

ATLANTA
Optical Fibers (1 of 5)
BCG, Round 2

Guidance for interviewer and


Problem statement narrative
information provided upon request
The client is a fiber optical manufacturer. (Fiber optics
are wires that transmit data)
The CEO of this fiber optical firm is new and is trying to - Data types transmitted over these wires are internet, TV,
assess the firm’s business situation. At a surface levels, phone, etc. .
she knows the firm is experiencing a 50% decline in
revenues, but has hired BCG to dig further. - The client specifically sells these wires to big firms like

use them to transmit data over long distances.


She has 3 specific questions:
-Why is the firm experiencing this revenue decline?
-When will the firm rebound?
-How does the firm relate to its competition?
Optical Fibers (2 of 5)
BCG, Round 2
Student should tackle each one of the three questions in this order:
The answer to this question is qualitative in nature. The industry as a whole is suffering because 4
1. Why is the firm years ago more optical fibers were put in the ground then their was demand for. Each fiber optical
experiencing this cable can support about 1 tera bit of information, and the demand per second (unit of time we’ll
revenue decline? use) is less than 1 tera bit per hour.

The student should be looking for data that will help them see when the market or demand for
these cables will come back.

When prompted, the interviewer will reveal:


2. When will the firm
-Candidate should use the market of fibers between D.C. and NY as representative of the world for
rebound?
the purposes of this case
-There are 1000 fibers between these two cities and the peak of demand is 100 terabits/sec.
-Every year the peak demand doubles. Use time zero of demand as 100 terabits, year 1 as 200 and
so on.

Demand will exceed capacity in year 4 so somewhere between year 3 and year 4, there fore more
Candidate should
cables will have to be installed and the company can rebound. This is sufficient for this part of the
determine that:
case (don’t need to get into how much of that market they can capture, etc.)
Optical Fibers (3 of 5)
BCG, Round 2
3. How does the firm Ask the candidate what they would look at to compare this firm to the competition. Don’t let them
relate to its go until they’ve exhausted most of the things on the list below. First have them list all things, then
competition? give them information on the firm as it relates to each category on their list.

�������� ����������� �� ����


����� C�������� ���� � ���� ����� �� �������� ����
������� ������� N�� �������� �� ���������, ��������� ������
M������������ C������� L������ ��������, ���� ���� ��������
C��� ��������� L����� ���� ���������/���� �������
����� �� M����� #1 �� ������, ���� ����� ��������

C������� B��� ���� �� ��� ����� ��������� �� ������ ���� A�&� ��� ������
G��������� F��������/D����������� ���� �� �����������
C���/A����� (B������ �����) L�� �� ���� �� ����, �� ����

Some of these are things you would always look at, but some of them are important to this case.
Candidate should For example, capacity matters because the firm with idle capacity will be able to grab the new
identify that: business when the market picks up again. Also the firm with cash on hand and no debt will be able
to get through tough times when the market is slow.
Optical Fibers (4 of 5)
BCG, Round 2

Tell the candidate that he/she’s done a good job getting through the 3 questions put forth, but you
After answering the 3
still have one more. Someone on the CEO’s team wants to get a quick sense of the firm’s variable
initial questions:
and fixed costs. This is an ad hoc question they want a quick  answer on.

Two months of data:

Provide this data: Month 1 Month 2


Produce 3MM km of fiber Produce 2MM km of fiber  
Costs are $50MM Costs are $40MM

1MM km fiber costs $10MM (difference between Month 1 & 2 costs)


Candidate should
determine that: Thus fixed costs are $20MM per month
Telecom (3 of 3)
BCG,
BCG, Roun
Round
d2

Exhibit A

������� % �� ����� % �� �������   ��������


D����� ����� 65% ���� �������� �� ��������, ������ �������� ������� ���
���� �� � ������� ���� O����� M�� ��� ���� ����� �� ���
I������� ����� 80 % ��������
Gas station (1 of 7)
BCG, Round 2

Guidance for interviewer and


Problem statement narrative
information
information provided upon request(

Your client is a gas company that operates in a town with a 1. What areare the proposed
proposed grocer
groceries
ies the gas station
station would
would now
population of 1,000. There is only one other
other gas company in this sell? (turn question around and ask the candidate).
town, and it is 1 mile away.
away. The other nearest gas
gas stations are
Likely answers include cigarettes, milk, snack foods;
outside town, and they are 20 miles away (see pict ure on next
probably not fresh produce or heal thy foods.
page).
2. Currently
Currently,, the gas
gas station
station is barely
barely scraping
scraping by.
by. Profit is
essentially 0.
Recently,
Recently, our client was approached by a supermarket with the
3. If asked anything about the other gas station, the answer is “we
idea of selling groceries in the gas station. Our client is a simple
don’t know, but assume they are identical”
businessman and has hired us to evaluate this proposal. What
should our client consider? Candidate should recognize that each gas station serves
500 people.

- 15
15 -
Gas station (2 of 7)
BCG, Round 2

Map of area (not to scale)

- 16
16 -
Gas station (3 of 7)
BCG, Round 2

Framework

Additional questions for candidate Solution guide

What information would you want to consider when • Revenue: Increased revenue from selling groceries i n store;
deciding whether or not to sell groceries at this gas station? more people coming to buy gas from this station instead of
(You
(You as interviewer are tryi ng to get candidate to provide station B.
this framework)

• Costs: Up front investment costs such as a freezer,


freezer, shelves,
etc.
• Recurring costs such as labor will be minimal since
same staff can handle gas and groceries.
• Will there be any profit-sharing with the grocery
store?

• Competition: What is keeping the grocery store from going


to gas station B as well?

- 17
17 -
Gas station (4 of 7)
BCG, Round 2

Costs

Additional questions for candidate Solution guide

What kind of costs could there be? • Investment in freezers, shelves, utilities.

• No increase in labor expected.

• Total increase in costs: one-time cost of $ 1.25MM.

- 18
18 -
Gas station (5 of 7)
BCG, Round 2

Revenue stream

Additional questions for candidate Solution guide

• Groceries: • People who shop here are probably not health conscious,
since gas station groceries are not healthy. They are
• What kind of people will shop for groceries here?
probably also in a rush, and will ma ke small purchases
• How much do they spend per year? (milk, cigarettes, jerky, etc.)
• Out of 1000 people, assume 500 will buy groceries
• What else can be a source of revenue? here at $200 per year per person

• Candidate should recognize that more people will • -> $100,000 per year
buy gas here now. • Increase in gas sales: very important distinction: people
won’t buy more gas, but more people will buy gas here
(stealing customers from station B)
• What is the total increase in sales?
• We can steal
steal 50% of station B’s
B’s customers (250
people), who purchase $100 in gas per year ->
• The gas station owner is a simple man and wants to look at $25,000 per year
this like a perpetuity.
perpetuity. What does this work out to?
• Total increase in sales per year: $12 5,000.
• (discount rate) r = 10%
• As a perpetuity: $1.25MM.
• (growth in cash flows) g = 0%

- 19
19 -
Gas station (6 of 7)
BCG, Round 2

Competition and Should they do it?

Additional questions for candidate Solution guide

What would you advise the gas station owner to do? • This is a prisoner’s
prisoner’s dilemma.
dilemma.

• If station A does it, it is value neutral, BUT,


BUT, we can take
$250,000 away from station B (good). But what is keeping keeping
station B from retaliating and doing the same thing?
Remember,
Remember, station A is barely scraping by. If station B
responds, we could go out of business since revenue would
no longer cover the one-time cost cost (bad). If station B does
not respond, then we can steal al l customers from station B
and be profitable (good).

• If A does not do it, station B might do it. Then station A


would lose $250,000 and it could go out of business (bad).

• No matter which scenarios the candidate chooses, push


hard for the other option and play devil’s
devil’s advocate.

- 20
20 -
Gas station (7 of 7)
BCG, Round 2

Final Recommendation

Recommendation Risks Next steps

(based on what candidate decides) (based on what candidate decides) Try to form an exclusive contract with the
grocery store.

Do it, and try to push station B out of Station B might respond and bring both
business. stations down.

Don’t do it, and hope station B doesn’t Or Station A might miss out on t he
do it either. opportunity and lose first mover
advantage.

- 21
21 -
Bidding on the Olympics (1 of 4)
BCG, Round 1

Guidance for interviewer and


Problem statement narrative
information provided upon request(1)

Our client, a major TV Network, wants to know how much to bid • This only includes the rights to U.S. programming, overseas
on the TV rights
rights for the 2016 Olympic Games. Bid will need to be programming is out of scope
paid in 2010.
• Assume they will only show the Olympics on their one flagship
channel
The amount of programming is as follows: • Olympic programming will replace regularly scheduled
programming
• Prime Time is considered anytime after 7pm on a weekday,
weekday,
16 days total
and all day during the weekends.
Opening Ceremonies on a Friday: 8pm  – 11pm
• Interviewee should point out that the network will earn ad
14 days of programming for 10 hours a day,
day, revenue while the coverage is on, and hopefully will consider
9am – 12 pm, 2pm – 5pm, and 7pm -11 pm M-F added viewership to the network as a whole due to Olympic
coverage.
11am – 9pm on the Weekends
• Interviewee should consider the costs to put on the event,
Closing Ceremony on the following Saturday from 8pm  – 11pm and opportunity cost of ad revenue from other program.

- 22
22 -
Bidding on the Olympics (2 of 4)
BCG, Round 1

Breakdown of Ad Revenue during Olympics

Prime Time Non Prime Time


Revenues $400k / 30 $200k / 30
seconds of ads seconds of ads

Amount of Ad time 10 min / hour 10 min / hour 

Breakdown of Costs

Prime Time
Cost associated with $428 MM
coverage*

Opportunity Cost of $1M / hour


ad revenue from other
programming

* This includes all fixed and variables costs for travel, equipment, salaries, etc. Don’t let the candidate get
caught up in these costs.

- 23
23 -
Bidding on the Olympics (3 of 4)
BCG, Round 1

Candidate should Revenues Programming hrs Minutes o f Ads Ad Revenue


calculate the total Prime Time 86 860 860*400K *2=$688M
revenue to be received Non Prime Time 60 600 600*200K*2=$240M Total Revenue
=$928 Million
in 2016.. The *2 is because you
make$400K per 30 seconds

Event Costs Opportunity Cost


$1 M per Total Cost Total Profit in 2016
$428M
… and the total costs programming =$574 Million =$928M -$574M
hour ($146M) = $354 Million

Now, the candidate should think about Time Value of Money

• Profit of $354 Million (Plus any additional bump for future viewership or required profit margin candidate may include) will be received in
2016, but bid must be paid in 2010.
• If asked, give them a Cost of Capital of 12%

• A great candidate will recognize rule of 72, which states that you divide 72 by the interest rate to determine how long (in years) it will
take an investment to double. 72 / 12 = 6, which is how long in the future we will receive our profits. Therefore, $354M in 2016 is worth
$177M in 2010.

- 24 -
Bidding on the Olympics (4 of 4)
BCG, Round 1

Answer Guidelines

• The key takeaways from this case are to:


• Recognize time value money
• Understand and identify Opportunity Costs
• Make judgment on unknown information (how much value having the Olympics wil l bring to other
programming on the network)
• Great candidates will:
• Understand there are 10 weekday days and 4 weekend days (plus the opening and closi ng) and not list
out every day to calc hours for programming
• Know and use the Rule of 72

• Exact answers are irrelevant, though should be around $177 million, with well thought out adjustments to
increase or decrease the number.

- 25 -
Household Cleaners Growth (1 of 6)
BCG Round 1

Guidance for interviewer and


Problem statement narrative
information provided upon request

Your client is a global consumer packaged goods company —Grime This case is about growth both through internal actions and
Co. through acquisition. Initially, the candidate should brainstorm an
array of possible growth strategies. Eventually, he or she will have
Grime Co. makes paper products (like paper towels), home to drill down on new products and acquisition, in addition to
cleaning products, and laundry care products. The company's considering market growth. Then, she or he will have to evaluate
Board of Directors has set an aggressive net sales target of $2 two targets, demonstrating an understanding of positive and
billion by 2015 (four years). Currently, net sales are at $1 billion. negative synergies. Without considering market growth, organic
The CEO has come to you to ask for help. growth, and inorganic growth —and without exploring synergies in
acquisition — the candidate will not be able to solve the case.
Specifically, our client would like you evaluate the company's
position and to help develop a strategy to deliver top-line results of Additional information:
$2 billion by 2015.
• The company has a strong stance on sustainability
Net sales: Retail sales minus trade spend. Trade spend is what • Sales are divided evenly between the t hree categories — 33%
manufactures pay distributors or retailers to incentivize them to • The company has low profit margins and does not want to
sell their products to end consumers. take on additional debt, so cash available for investment is
about $300 million. (Be sure not to say “for acquisition”.)

If the candidate asks which growth strategies Grime Co. has


considered, the interviewer should prompt her or him to
brainstorm various options — see next slide.

- 55 -
Household Cleaners Growth (2 of 6)
BCG Round 1

The candidate must touch on market growth, new products, and acquisitions to solve the case — in any order. The
following structure is how the candidate may organize information. Profit or cost should not be part of the structure.

Area the candidate should explore: Information provided upon request


Market growth: Growth through maintaining When the candidate asks, reveal that market growth alone will bring sales to $1.5
market share in a growing market billion by 2015. Specifically, the company is growing overall at 10% and expects to
1 maintain a constant market share. (10% compounded over four years is roughly
$500 million incremental.) In the interest of keeping this case shorter, the
candidate does not have to calculate this. If the candidate asks about categories,
tell him or her that growth is about the same in all three.

Organic growth: Actions taken within the The candidate must cite new products: it is the only organic growth strategy that
organization to drive revenue. Examples: is viable for our client in this case. The interviewer should provide logical reasons
to why the other options are not available at this time.
• Price adjustments to drive volume
• Increased advertizing Our client has a new toilet cl eaning product in development that analysts believe
• Expansion into new geographies will do well. The following details should be provided by request:
2 • Vertical integration
• Promotions and deals • Product is near launch — hits shelves in a months
• Negotiation for better placement • Price will be $5 a unit, but requires 20% trade spend per product to reach
• New products volume target
• Expected to sell 40 million units on average per year
Interviewer should steer candidate to explore
new products No other investment is required — sunk cost. (See slide 3 for calculations.)

Inorganic growth: Growth through acquisition The candidate must identify growth through acquisition: Our client ’s Corporate
3 or joint venture Development department has identified two high-priority acquisition targets —
Organoclean and Home Defense Inc. (See slide 4 for detailed information.)
- 56 -
Household Cleaning Services (7 of 8)
BCG, Mock interview

Should they enter the market? (cont.)

Additional questions for candidate Solution guide

Anything else to consider before making your final • Candidate should come up with ways to separate
recommendation, such as how to separate ourselves from Restoration Co. from its competition:
the competition?
• Using premium cleaning supplies (such as
sustainable chemicals, scented shampoo, etc.)
• Performing a survey of the house to assess for
potential fire or flood hazards.
• Offer a free cleaning first.
• Bad idea: enter into a pricing war in a segment that
competes on quality.

- 67 -
Household Cleaning Services (8 of 8)
BCG, Mock interview

Final Recommendation

Recommendation Risks Next steps

Enter the market: Market size is $50B 27% margin needs to be put in context of Review financing and up-front costs
(even better: $5B), and margins are a current business
healthy 27%, since we will compete on
quality with the national players. How can we use our current client book
Possible brand dilution to jump start business?
- But let’s offer only quality products
How will our sales model change
(currently relies upon insurance
Need to separate our business
companies, will change to door to door
- But go with ideas mentioned on the marketing)?
slide before

- 68 -
Bio-Product Growth (1 of 7)
BCG, Round 1

Guidance for interviewer and


Problem statement narrative
information provided upon request(

Your client is a chemical company, ChemCo. Due to stagnant 1. Last year bio-products segment accounted for $300 million in
growth in the chemicals segment, they decided to create a high revenue
growth bio-products segment. This segment sells these products 2. Bio-products industry has been growing at 10%, hence the
to universities and labs primarily for use in drug development. CEO’s target
Revenue for the new segment has been growing at 3 -5% per
annum for the last few years. The CEO is unhappy with this and
wants to achieve 10% growth in revenue in the next year. You have
been hired to help bridge this gap and ac hieve this target.

A good structure will include:


Industry (growth and trends)
Client (growth and trends, limitations on growth, product mix)
Growth Strategies (geographies, channels, acquisitions, product
mix, R&D)

- 83 -
Bio-Product Growth (2 of 7)
BCG, Round 1

Framework

Additional questions for candidate Solution guide

After the structure, when the candidate asks for • Some implications of Exhibit 1 include:
Industry/competitor information provide Exhibit 1. • ChemCo is “over-diversified”
• Could have achieved diseconomies of scope
Allow the candidate to make some insights, push them if • Scale operations could be more efficient
necessary. Eventually get them to consider the use of R&D. • Inefficient use of resources:
What are the implications of R&D from that slide? • Sales and Marketing
• Research and Development
Give them Exhibit 2. • Some implications of Exhibit 2 include:
• ChemCo is below market trend
• C7, C5 & C3 could be studied to understand efficient
R&D
• Inefficient use of R&D funds
• Could eliminate inefficient R&D or spend more due
to high correlation between R&D and revenue

- 84 -
Li-Ion Battery Separators (4 of 7)
BCG, Round 1
Customer Supply Base

100%

90%

80%

70% C7

60% C6

C5
50%
C4
40%
C3
30% C2

20% C1

10%

0%
Sony LCG Sanyo Other

• Customers generally have a primary supplier and a secondary supplier


• Industry characterized by strong supplier relationships given safety concerns

- 93 -
Li-Ion Battery Separators (5 of 7)
BCG, Round 1

Battery Component Prices

2000 2010
Cathode $8 $4
Anode $2 $1
Separator  $3 $2
Other  $7 $3
TOTAL $20 $10

Note: Costs have decreased at the same rate for all battery components.

- 94 -
Li-Ion Battery Separators (6 of 7)
BCG, Round 1

Additional questions for candidate Solution guide

A good analysis will consider the following:


So, should we enter the Li-Ion Battery Separator Market? (The
• Green Field:
candidate must realize that this question must also cover
• Pros – Management Control
“how should we enter”. If they do not go down this route,
• Cons – Expensive and timely
direct them with the additional questions.)
• Joint Venture:
• Pros – Easier and quick
• Cons – Limited control, finding partner
• What are the possible ways to enter?
• Acquisition:
• What are the pros and cons of each?
• Pros – Quicker, Management Control
• Cons – Culture clashes, buyer’s curse (overpaying)
• License/Sell
• Pros – Easier, less commitment to new market (get your milk
without the cow)
• Cons – finding buyers

- 95 -
Li-Ion Battery Separators (7 of 7)
BCG, Round 1

• Our client should License the technology due to a lack of knowledge and experience in the Li-Ion Battery
Sample Separator market, existing buyer-supplier relationships, and the expensive certification process inherent in the
industry.
Recommendation

• No industry experience
A solid interview will • Existing manufacturers will not want to change technology
address other • No discussion on benefits of new technology
potential risks…

• Understand technology and value proposition to manufacturers


And suggest next • Patent technology and determine licensing fee
• Identify interested manufacturers and develop interest from customers (Sony, LCG, Sanyo, etc.)
steps…

- 96 -
Case: Retirement Apartment Complexes
Potential Categories of Candidate’s Framework
88

Notable comments / potential discussion points


Most candidates will tend to examine revenues and costs once they realize this case is about maximizing profitability. While this is
the eventual course of action it is important for the candidate to understand the current markets that the client operates in and glean
key learnings. Remember – if the candidate had asked the financial goal question, the client wants to maximize profitability through
new or EXISTING properties.

Potential points include, but are not limited to:

n ustry ana ysis


firm position / market share

trends

competition

Profit drivers
• revenue
• costs
Case: Retirement Apartment Complexes
Potential Issue Tree & Approach to Solving the Case
89

Key elements of analysis to solve the case

Market Size Revenue Cost


When dealing with apartment Identify sources of revenue from Identify cost buckets and distinguish fixed
complexes, examine the following: apartment complexes: from variable components

• Number of properties • Rent • Maintenance


• Number of units er ro ert • Utilities if mana ed b buildin co. • Amenities
• Occupancy Rates • Premium for amenities, if any • Leasing/Marketing
• Retail revenue, if any • Utilities
• Maintenance or other fees • Property Taxes

Possible follow-up and guidance Possible follow-up and guidance Possible follow-up and guidance to
to interviewer to interviewer interviewer
If the candidate has not yet asked for Recall that the Chicago property pilot
the current apartment complexes in the provides residents with additional
Southern states, give her/him details: amenities compared to properties in the
• Competition is strong in the south; south. The candidate should examine this
easy to build new apartments. aspect to accurately compare costs.
• Occupancy rates typically start out at
90%+ but eventually settle at 80% as
new complexes are built.
• 3-5% growth every year fed by new
people who migrate to the south
Case: Retirement Apartment Complexes
Question 1 – Math
90

Math Question: Which is more profitable – north or south?


1. First compute occupancy levels and revenue. For this discussion all apartments in a given region can be considered equivalent.

South: 800 units/bldg * 0.8 occupancy rate * $500 rent/mo = $320,000 per mo (let candidate know that rent includes
maintenance, amenities, utilities; each property generates identical revenue so leave calculations on an individual property basis)

North: 400 units * 0.9 occupancy rate * $1,000 rent/mo = $360,000 per mo
Rent includes amenities charges for a 24 hr. on-site medical facility that residents benefit from in a huge way

2. Next compare costs (per month)

Cost Item North South


Maintenance 400 units * $200 = $80,000 800 units * $125 = $100,000
Amenities $54,000 $48,000
Utilities $45,000 $38,000
SG&A $60,000 $80,000
Medical Facility $60,000 $0

Total Cost per mo $299,000 $266,000


Total Revenue per mo $360,000 $320,000
Profit per mo per bldg $61,000 $54,000
Case: Retirement Apartment Complexes
Question 1 – Math, cont’d
91

Math Question cont’d…


3. Next calculate profitability for both locations by using the formula
Profit per mo per building
÷
Total Revenue per mo
4. Turns out that the south and north are equally profitable at 16.8% approx.

Overall approach, good shortcuts & solution Information to provide up front


The key to this question is not to stop at calculating profit and N/A
jump to a conclusion then. A good candidate will take the next
step to calculate profitability and realize that the South and
the North are equally profitable relative to revenue
generated.

Now the candidate should consider the profitability relative to


the cost to implement various initiatives. Is it more expensive to Provide information if asked
open in the North or South? What is the incremental cost of
At this point the candidate may ask about the market
constructing medical facilities in the south (relative to increased
size in the North or the cost to open new facilities in the
earnings)? What is the cost of enhancing some amenities in the
North (or the cost to renovate / alter facilities in the
South to generate higher rent? Higher NPV is the goal.
South). There is no additional data available, but for the
purposes of this discussion we can assume that the return
We will not explore the capital cost in this case, but a return on
relative to cost is also comparable in North vs. South.
cost analysis should be part of next steps.
China Outsourcing Opportunity  –  Solution
52

Calculate the Costs of Outsourcing in China

Math Question Looks like they will save 25% of costs by going to China

Costs in Costs in China relative Costs in


Costs U.S. ($/lb) to U.S. China ($/lb)

Labor 0.30 8% of wage rate =.3*(10/8)*(8/100) = .03


80% of productivity
Material
Plastic resin
• 0.30 80% .24
Other material (incl. packaging)
• 0.20 75% .15

Variable overhead 0.05 140% .07

Fixed overhead 0.10 60% .06

Transportation
China to U.S. distribution center
• N/A $6K to ship 40K lbs. =6/40 = .15
U.S. distribution center to customer
• 0.05 Same .05

Total 1 00 75
Solution element  –  recommendation et al.
53

Recommend outsourcing the paper and plastic plates to China. Keeping the other lines in the US due to
quality and trend issues.
Recommendation

Change in fixed overhead costs currently if move production to China. Additional mfg capacity  –  other
Risks product lines, rent out to another company, close certain lines; PR / Brand image risks

Research risks mentioned above to determine whether beneficial to implement


Next Steps

BONUS
What might be some other benefits?

- Potentially easy access into the growing Asian


economies in the plastic market.
Establishing the case
70

Case7 : Jamaican Land Investment


BCG: Round 1

Problem statement narrative Information to be given upon request (cont‟d)

Our client is thinking about buying a piece of land in •Fixed Cost: $500 initial set up (first year only), $350 per year
Jamaica for $3000 and has asked us to determine for salaried labor
whether or not this is a good idea. •Market Demand per year
• 5000 Trees, 1000 shrubs, 1000 fruit, 2500 exotic
flowers
Information to be given only upon request •Penetration rate: competitors cannot meet current demand.
•The price of the land is $3000
• Competitors have 60% tree share, 20% shrub
•Total acreage: 10 acres
share, 85% fruit share, 90% exotic flower
•Financial Target: $4,500 profit within first two years, excluding
share…the remaining shares can be captured by
$3000 purchase price
our client.
• When prompted about use of land, ask candidate to brainstorm
•How many plants can fit on an acre? When asked about how
possibilities before giving him/her the answer: real estate
much of each plant can fit onto an acre, throw the question back
development, farming, hold and sell it once it appreciates, etc.)
and ask: “which plants do you think would have less of per acre?”
•Land will be used for agriculture
(trees and shrubs b/c they take up a lot of room)
• Trees, Shrubs, Fruit, Exotic Flowers
• 10 Trees/acre
•Cannot mix products (trees and shrubs) on same acre. Only one
• 25 Shrubs/acre
type of plant allowed per acre.
• 75 Fruit/acre
•Price per plant
• 50 Exotic Flowers/acre
• Tree $50, Shrub $35, Fruit $15, Exotic $25
•Variable Cost per plant
• Tree $30, Shrub $25, Fruit $11, Exotic $17
Case Sequence
71

Candidate may propose analysis / action in:

Market Size/Est. Demand Margins Per Unit Profitability Per Acre


Demand P-C=Profit per unit # of units per acre x profit per unit
 Market Size x Client Penetration Rate $50-$30= $20 per Tree 10 trees per acre x $20= $200/acre
•5000 trees x 40%= 2000 units $35-$25= $10 per Shrub 25 shrubs per acre x $10= $250/acre
•1000 shrubs x 80%= 800 units $15-$11= $4 per Fruit 75 fruit per acre x $4= $300/acre
•1000 fruit x 15%= 150 units $25-$17= $8 per Exotic 50 exotic flowers per acre x $8=
•2500 exotic x 10%= 250 units $400/acre

Supply
*Client‟s capability to meet estimated
demand is dependent on the number of
acres it has (10 acres).

Possible follow-up and Possible follow-up and Possible follow-up and


guidance to interviewer guidance to interviewer guidance to interviewer
Go From Most Profitable To Least 5 acres of Exotic Flowers x $400/acre= To make the case more challenging, ask:
•Exotic Flowers: 250 units demanded/50 $2000 -How would you price each unit? (cost-
units per acre= 5 acres used fpr Exotic 2 acres of fruit x $300= $600 based pricing, look at competitors prices,
Flowers 3 acres of shrubs x $250= $750 price by segment (premium supermarkets vs.
•Fruit: 150 units demanded/75 units per fruit stands, etc.)
acre= 2 acres used for Fruit  Y1: $3,350-500-350 = $2,500 (do not
•Shrubs: 800 units demanded/ 25 units per include $3000 investment) -If demand was there, would you use all 10
acre= all remaining acres (3 acres used for Y2: $3, 350-$350= 3000 acres for exotic flowers? Why or why not?
Shrubs) (important to diversify products!)
Total # of acres should add to 10! Total Profit For First Two Years: $5,500
Interviewee’s Solution (cont.):
Q: Knowing that new drugs are always being developed, I’d want to evaluate those that are currently
in the pipeline of our competitors and what we anticipate the impact of them being on the market at
the time of launch. You can find this information in publications like R&D directions as well as other
trade publications.
 A: Sounds good. Besides volume what else should you be thinking about?
Q: Pricing. Pricing for drugs has a lot to do with demonstrated efficacy as well as whether you are the
first drug in a given class to be launched. Other things that you might want to consider include
convenience (dosing requirements) and other drugs used to treat the therapy even if the mechanism of
action differs.
 A: Great. What about costs?
Q: From a cost perspective, I’d want to consider both fixed and variable costs. This would include
any new facilities that need to be built to manufacture the drug as well as the R&D and sales and
marketing launch costs associated with the product.
 A: And what would you do with this information?
Q: I’d determine the free cash flow number for each year and then discount that back to the current
 period. Additionally, since the process has high attrition, I’d also want to adjust for the probability of
success at each phase.
 A: Great. Let’s say you’ve done all this and it turns out that ½ the compounds in pre-clinical
 development have no value. How are you going to plug the hole from a revenue perspective?

Q: Any time you are considering product development there are two options – make or buy. Under
the make category, we could try to expedite the development of existing pipeline drugs to address the
gap but this is fairly unlikely to prove successful given the highly complex development process.
 A: What else could address the revenue considerations if you can’t expedite development?
Q: You could work with your existing portfolio of marketed products to increase the amount of
• Before doing anything else, the candidate sets out a clear road map for tackling the question –
“I am going to do an NPV which will mean looking at revenues and costs”
• The candidate clarifies the expectations of the interviewer – “Should we be looking at
generalities or specifics?” The last thing you want to be doing in a case interview is giving a
great answer that bores the interviewer because it is not the aspect of the question that they
were interested in.
• The candidate has a clear and logical approach to sizing the market for each drug. Not only are
the steps sensible and meaningful but they are also readily verifiable by available data (such as
American Cancer Society data on the number of patients with a particular cancer condition). It
is quite common for interviewers to ask how you would go about getting the data to support a
certain step in you logic so it is important to be choosing divisions where this information is
likely to be available.
• The candidate does not get bogged down when the interviewer changes the question. If you are
asked a new question, like how to fill a hole in the pipeline, then develop a new framework for
approaching this aspect – “there are two options to consider, make or buy”

Similar techniques could be applied to almost any market sizing question.


CASE 17: ELECTRONICS RETAILER (BCG – Round 2)

Your client is a major consumer electronics retailer who sells A/V equipment, computers,
content (CDs, DVDs), and software. Recently, the store has seen increased competition from
bricks and mortar stores like Wal-Mart as well as online retailers like Dell. Your client has 3
questions:
1) How is the industry evolving?
2) What is the winning retail model?
3) What should we do?

Additional Information Provided After Relevant Questions:

•  None provided

Interviewee’s Solution:

Q: To answer these three questions, I’d like to look in 3 areas. First I’d like to know more about the
current market including understanding customers, the products they are buying, and how these have
changed over time. Next, I’d like to take a look at the different types of competitors that are affecting
our company and our firm’s profitability. Lastly, I’d like to know more about our client in terms of
their business model, including core competencies, strategic direction, and their store model, to better
understand what options are realistic. Is there a particular, place you would like me to start?
 A: Before we get started, can you tell me what you think makes a su ccessful retail model?
Q: Sure. I think there are several elements. First, I think that a retailer needs to have a clear
understanding of their customer base. This will drive two things: having the appropriate product mix
to serve them and serving them through the appropriate distribution channel (online vs brick and
Interviewee’s Solution (cont.):
Q: From the perspective of our client, CDs and DVDs are not going to be a substantial part of their
 business going forward because the ability to buy content directly from producers removes the
middleman. Our client will more directly need to rely on other parts of its product mix in order to be
successful. In the interim, it is critical that our client manage the transition carefully in order to
maximize the opportunity associated with this closing window which will likely last for several more
years.
 A: Agreed. So, let’s assume that we did some additional analysis and it turns out that our client
 needs to double the volume of its other products to counteract this. Fixed costs don’t grow in
 proportion to sales – they grow on a 75% scale curve meaning that even though sales double now
 FC are only 75% of what they were on a % of revenue basis. Our gross margin is 25% and net
income is 0%. What would our net income be if we doubled volume?
Q: Let’s say that Year 1 revenue was $100 making operating profit 25 and fixed costs 25. In Year 2,
we would make $200 and our operating profit would be $50. Given the scale curve, our FC should be
about $38 giving us net income of $12 or 6%.
 A: Exactly.

Commentary/Recommendations:

This is a fantastic sample case because the dialogue provided by the candidate demonstrates exactly
what the case interview experience can end up being like. Most importantly it demonstrates exactly
how that experience can differ wildly from what you were originally expecting, and what the case
interview books tell you to expect.

At the outset of the discussion, the candidate develops a framework for answering the question. All
they have to do now is walk through that framework right?
• The interviewer then chooses not to let you walk through the framework, either because
they are bored with having asked the same question ten times already that day or because
they think they doing so will give them very little extra insight into your thinking beyond
what they already got from hearing what your framework was going to be.
• Instead they choose to test you more deeply by asking you a series of questions that give
them more insight into your ability to really understand and think about a business
 problem or market.
• Far from being a sign of failure, getting grilled with questions in this fashion may
actually be a sign that you are doing well.

 Note, there is a fundamental difference between interviewers asking you questions to prompt
you because you don’t seem to know what you are doing and interviewers asking you
questions (as in this case) because they are really testing you.

Don’t assume you can sit back and let them ask you the questions they are interested in. You
cannot. You are always responsible for driving the case and you need to set out a framework
and start working your way logically through it. If however, at some stage during this process,
the interviewer starts firing questions at you then you need to be able to work with it.

Answer their questions as insightfully as you can and let them take you where they want to go.
Think about what things would be like in the real world – How would the CD market be
different to the DVD market? You must have had plenty of experiences with both so apply
some common sense and you are more than half way to a decent answer.

If the questions run out, then tie everything back in to your original framework and keep going.
But more times than not the questions will only run out when they bring you to a case
CASE 20: HOTEL (POST 9-11) (BCG – Round 1)

BCG has been hired after 9/11 to look into quantifying the amount of damage to revenue
streams that has occurred in the Manhattan hotel business. How would you go about
determining / quantifying the impact, and how would you structure a recovery plan?

Additional Information Provided After Relevant Questions:

• Hotels use a metric called “Revpar” which is equal to the average daily room rate * % occupancy.
Revpar was $200 before 9/11, $125 for the remainder of 2001, $150 for 2002, and $175 for 2003.
• I was asked to assume that if 9/11 had not occurred that the Revpar would have been flat for the
entire time period.
• I was told that there were 100,000 hotel rooms in Manhattan. I was then asked to quantify the
revenue impact.

Interviewee’s Solution:

• I structured my discussion by segmenting the market into US business travelers, international


 business travelers and leisure travelers. I also tried to make a separation between individual hotels
and chain hotels, but it was clear that the case giver was only interested in looking at things on an
aggregate basis.

• The key, I think, to doing well in this case is to focusing not only on quantifying the damage, but
also to coming up with creative suggestions for a recovery plan.

You might also like